Методом гаусса как решить: Решение систем линейных уравнений методом Гаусса

Содержание

Метод Гаусса — примеры c решением, теоремы и формулы

Метод Гаусса – идеальный вариант для решения систем линейных алгебраических уравнений (далее СЛАУ). Благодаря методу Гаусса можно последовательно исключать неизвестные путём элементарных преобразований. Метод Гаусса – это классический метод решения СЛАУ, который и рассмотрен ниже.

Карл Фридрих Гаусс – немецкий математик, основатель одноименного метода решения СЛАУ

Карл Фридрих Гаусс – был известным великим математиком и его в своё время признали «королём математики». Хотя название «метод Гаусса» является общепринятым, Гаусс не является его автором: метод Гаусса был известен задолго до него. Первое его описание имеется в китайском трактате «Математика в девяти книгах», который составлен между II в. до н. э. и I в. н. э. и представляет собой компиляцию более ранних трудов, написанных примерно в X в. до н. э.

Метод Гаусса – последовательное исключение неизвестных. Этот метод используется для решения квадратных систем линейных алгебраических уравнений.

Хотя уравнения при помощи метода Гаусса решаются легко, но всё же студенты часто не могут найти правильное решение, так как путаются в знаках (плюсы и минусы). Поэтому во время решения СЛАУ необходимо быть предельно внимательным и только тогда можно легко, быстро и правильно решить даже самое сложное уравнение.

У систем линейных алгебраических уравнений есть несколько преимуществ: уравнение не обязательно заранее на совместность; можно решать такие системы уравнений, в которых число уравнений не совпадает с количеством неизвестных переменных или определитель основной матрицы равняется нулю; есть возможность при помощи метода Гаусса приводить к результату при сравнительно небольшом количестве вычислительных операций.

Определения и обозначения

Как уже говорилось, метод Гаусса вызывает у студентов некоторые сложности. Однако, если выучить методику и алгоритм решения, сразу же приходит понимание в тонкостях решения.

Для начала систематизируем знания о системах линейных уравнений.

Обратите внимание!

СЛАУ в зависимости от её элементов может иметь:

  1. Одно решение;
  2. много решений;
  3. совсем не иметь решений.

В первых двух случаях СЛАУ называется совместимой, а в третьем случае – несовместима. Если система имеет одно решение, она называется определённой, а если решений больше одного, тогда система называется неопределённой.

Метод Крамера и матричный способ не подходят для решения уравнений, если система имеет бесконечное множество решений. Вот поэтому нам и нужен метод Гаусса, который поможет нам в любом случае найти правильное решение. К элементарным преобразованиям относятся:

  • перемена мест уравнений системы;
  • почленное умножение обеих частей на одно из уравнений на некоторое число, так, чтобы коэффициенты при первой переменной в двух уравнениях были противоположными числами;
  • сложение к обеим частям одного из уравнений определённых частей другого уравнения.

Итак, когда мы знаем основные правила и обозначения, можно приступать к решению.

Теперь рассмотрим, как решаются системы методом Гаусса на простом примере:

   

где а, в, с  – заданные коэффициенты, d – заданные свободные члены, x, y, z – неизвестные. Коэффициенты и свободные члены уравнения можно называть его элементами.

Если = = = , тогда система линейных алгебраических уравнений называется однородной, в другом случае – неоднородной.

Множественные числа , , называются решением СЛАУ, если при подстановке , , в СЛАУ получим числовые тождества.

Система, которую мы написали выше имеет координатную форму. Если её переделать в матричную форму, тогда система будет выглядеть так:

– это основная матрица СЛАУ.

– матрица столбец неизвестных переменных.

– матрица столбец свободных членов.

Если к основной матрице добавить в качестве – ого столбца матрицу-столбец свободных членов, тогда получится расширенная матрица систем линейных уравнений. Как правило, расширенная матрица обозначается буквой , а столбец свободных членов желательно отделить вертикальной линией от остальных столбцов. То есть, расширенная матрица выглядит так:

Если квадратная матрица равна нулю, она называется вырожденная, а если – матрица невырожденная.

Обратите внимание!

Если с системой уравнений:          

Произвести такие действия:

  • умножать обе части любого из уравнений на произвольное и отличное от нуля число ;
  • менять местами уравнения;
  • к обеим частям любого из уравнений прибавить определённые части другого уравнения, которые умножаются на произвольное число ,

тогда получается эквивалентная система, у которой такое же решение или нет решений совсем.

Теперь можно перейти непосредственно к методу Гаусса.

Простейшие преобразования элементов матрицы

Мы рассмотрели основные определения и уже понимаем, чем нам поможет метод Гаусса в решении системы. Теперь давайте рассмотрим простую систему уравнений. Для этого возьмём самое обычное уравнение, где и используем решение методом Гаусса:

Из уравнения запишем расширенную матрицу:

Из данной матрицы видно, по какому принципу она записана. Вертикальную черту не обязательно ставить, но просто так удобнее решать систему.

Определение

Матрица системы – это матрица, которая составляется исключительно с коэффициентами при неизвестных. Что касается расширенной матрицы системы, так, это такая матрица, в которой кроме коэффициентов записаны ещё и свободные члены. Любую из этих матриц называют просто матрицей.

На матрице, которая написана выше рассмотрим, какие существуют элементарные преобразования:

1. В матрице строки можно переставлять местами. Например, в нашей матрице спокойно можно переставить первую и вторую строки:

.

2. Если в матрице имеются (или появились) пропорциональные строки (одинаковые), тогда необходимо оставить всего лишь одну строку, а остальные убрать (удалить).

3. Если в ходе преобразований в матрице появилась строка, где находятся одни нули, тогда такую строку тоже нужно удалять.

4. Строку матрицы можно умножать (делить) на любое число, которое отличное от нуля.

Такое действие желательно проделывать, так как в будущем проще преобразовывать матрицу.

5. Сейчас рассмотрим преобразование, которое больше всего вызывает затруднение у студентов. Для этого возьмём изначальную нашу матрицу:

Для удобства умножаем первую строку на (-3):

Теперь ко второй строке прибавляем первую строку, которую умножали на -3. Вот что у нас получается:

В итоге получилось такое преобразование:

Теперь для проверки можно разделить все коэффициенты первой строки на те же и вот что получается:

В матрице верхняя строка преобразовалась:

Первую строку делим на и преобразовалась нижняя строка:

И верхнюю строку поделили на то же самое число :

Как вы можете убедиться, в итоге строка, которую мы прибавляли ни капельки не изменилась, а вот вторая строка поменялась. ВСЕГДА меняется только та строка, к которой прибавляются коэффициенты.

Мы расписали в таких подробностях, чтобы было вам понятно, откуда какая цифра взялась. На практике, например, на контрольной или экзамене матрица так подробно не расписывается. Как правило, в задании решение матрицы оформляется так:

.

Обратите внимание!

Если в примере приведены десятичные дроби, метод Гаусса в этом случае также поможет решить систему линейных алгебраических уравнений. Однако, не стоит забывать, что следует избегать приближённых вычислений, так как ответ будет неверным. Лучше всего использовать десятичные дроби, а от них переходить к обыкновенным дробям.

Алгоритм решения методом Гаусса пошагово

После того, как мы рассмотрели простейшие преобразования, в которых на помощь пришёл метод Гаусса, можем вернуться к нашей системе, которую уже разложили по полочкам и пошагово распишем:

Шаг 1. Переписываем систему в виде матрицы

Записываем матрицу:

Шаг 2. Преобразовываем матрицу: вторую строку в первом столбце приводим к нулю

Как мы привели вторую строку в первом столбце к нулю описано выше.

Напомним, что первую строку умножали на и вторую строку прибавили к первой , умноженной на .

Шаг 3. Приводим матрицу к ступенчатому виду

Теперь вторую строку можно поделить на 2 и получается:

Верхнюю строку делим на и приводим матрицу к ступенчатому виду:

Когда оформляют задание, так и отчёркивают простым карандашом для упрощения работы, а также обводят те числа, которые стоят на “ступеньках”. Хотя в учебниках и другой литературе нет такого понятия, как ступенчатый вид. Как правило, математики такой вид называют трапециевидным или треугольным.

Шаг 4. Записываем эквивалентную систему

После наших элементарных преобразований получилась эквивалентная система:

Шаг 5. Производим проверку (решение системы обратным путём)

Теперь систему нужно решить в обратном направлении, то есть обратным ходом, начиная с последней строки.:

находим : ,

,

.

После находим :

,

.

Тогда:

.

Как видим, уравнение решено правильно, так как ответы в системе совпадают.

Решение систем линейных уравнений методом Гаусса, в которых основная матрица невырожденная, а количество в ней неизвестных равняется количеству уравнений

Как мы уже упоминали, невырожденная матрица бывает тогда, когда . Разберём систему уравнений невырожденной матрицы, где уравнений по количеству столько же, сколько и неизвестных. Эту систему уравнений решим другим способом.

Дана система уравнений:

Для начала нужно решить первое уравнение системы относительно неизвестной переменной . Далее подставим полученное выражение сначала во второе уравнение, а затем в третье, чтобы исключить из них эту переменную.

Теперь переходим ко второму уравнению системы относительно и полученный результат подставим в третье уравнение.. Это нужно для того, чтобы исключить неизвестную переменную :

Из последнего, третьего уравнения мы видим, что . Из второго уравнения находим . И последнее, находим первое уравнение .

Итак, мы нашли все три неизвестных при помощи последовательного исключения. Такой процесс называют – прямой ход метода Гаусса. Когда последовательно находятся неизвестные переменные, начиная с последнего уравнения, называется обратным ходом метода Гаусса.

Когда выражается через и в первом уравнении, а затем подставляется полученное выражение во второе или третье уравнения, тогда, чтобы привести в к такому же результату, необходимо проделать такие действия:

  • берём второе уравнение и к его левой и правой частям прибавляем определённые части из первого уравнения, которые умножаются на ,
  • берём третье уравнение и к его левой и правой частям прибавляем определённые части из первого уравнения, которые умножаются на .

И действительно, благодаря такой процедуре у нас есть возможность исключать неизвестную переменную со второго и третьего уравнения системы:

Возникают нюансы с исключением неизвестных переменных тогда, когда в уравнении системы нет каких-либо неизвестных переменных. Рассмотрим такую систему:

В этой системе в первом уравнении нет переменной и поэтому у нас нет возможности решить первое уравнение системы относительно , чтобы исключить данную переменную из остальных уравнений. В таком случае выход есть. Нужно всего лишь уравнения переставить местами.

Так как мы описываем уравнения системы, в которых определитель основных матриц отличен от нуля, тогда всегда есть такое уравнение, в котором есть необходимая нам переменная и это уравнение мы можем поставить туда, куда нам нужно.

В примере, который мы рассматриваем, достаточно всего лишь поменять местами первое и второе уравнение.

Теперь мы можем спокойно разрешить первое уравнение относительно переменной и убрать (исключить) из остальных уравнений в системе. Вот и весь принцип работы с такими, на первый взгляд, сложными системами.

Решение систем линейных уравнений методом Гаусса, в которых основная матрица вырожденная, а количество в ней неизвестных не совпадает с количеством уравнений

Метод Гаусса помогает решать системы уравнений, у которых основная матрица прямоугольная или квадратная, но основная вырожденная матрица может совсем не иметь решений, иметь бесконечное множество решений или иметь всего лишь одно единственное решение.

Рассмотрим, как при помощи метода Гаусса устанавливается совместность или несовместность систем линейных уравнений. В случае, если есть совместность определим все решения или одно решение.

В принципе, исключать неизвестные переменные можно точно так, как описано выше. Однако, есть некоторые непонятные ситуации, которые могут возникнуть в ходе решения:

1. На некоторых этапах в момент исключения неизвестных переменных некоторые уравнения могут обратиться в тождества . В данном случае такие уравнения лишние в системе и их можно смело полностью убирать, а затем продолжать решать уравнение методом Гаусса.

Например, вам попалась подобная система:

У нас получается такая ситуация

Как видим, второе уравнение . Соответственно, данное уравнение мы можем из системы удалить, так как оно без надобности.

Дальше можно продолжать решение системы линейных алгебраических уравнений уравнений традиционным методом Гаусса.

2. При решении уравнений прямым ходом методом Гаусса могут принять не только одно, но и несколько уравнений такой вид: , где – число, которое отличное от нуля. Это говорит о том, что такое уравнение никогда не сможет превратиться в тождество даже при любых значениях неизвестных переменных. То есть, можно выразить по-другому. Если уравнение приняло  вид, значит система несовместна, то есть, не имеет решений. Рассмотрим на примере:

Для начала необходимо исключить неизвестную переменную из всех уравнений данной системы, начиная со второго уравнения. Для этого нужно прибавить к левой и правой частям второго, третьего, четвёртого уравнения части (левую и правую) первого уравнения, которые соответственно, умножаются на (-1), (-2), (-3). Получается:

В третьем уравнении получилось равенство . Оно не подходит ни для каких значений неизвестных переменных , и , и поэтому, у данной системы нет решений. То есть, говорится, что система не имеет решений.

3. Допустим, что при выполнении прямого хода методом Гаусса нам нужно исключить неизвестную переменную , и ранее, на каком-то этапе у нас уже исключалась вместе с переменной . Как вы поступите в таком случае? При таком положении нам нужно перейти к исключению переменной . Если же  уже исключались, тогда переходим к ,  и т. д.

Рассмотрим систему уравнений на таком этапе, когда уже исключилась переменная :

Такая система уравнений после преобразования выглядит так:

Вы наверное уже обратили внимание, что вместе с исключились и . Поэтому решение методом Гаусса продолжаем исключением переменной из всех уравнений системы, а начнём мы с третьего уравнения:

Чтобы завершить уравнение прямым ходом метода Гаусса, необходимо исключить последнюю неизвестную переменную из последнего уравнения:

Допусти, что система уравнений стала:

В этой системе нет ни одного уравнения, которое бы сводилось к . В данном случае можно было бы говорить о несовместности системы. Дальше непонятно, что же делать? Выход есть всегда. Для начала нужно выписать все неизвестные, которые стоят на первом месте в системе:

В нашем примере это , и . В левой части системы оставим только неизвестные, которые выделены зелёным квадратом а в правую перенесём известные числа, но с противоположным знаком. Посмотрите на примере, как это выглядит:

Можно придать неизвестным переменным с правой части уравнений свободные (произвольные) значения: , , , где , ,  – произвольные числа.

Теперь в правых частях уравнений нашей системы имеются числа и можно приступать к обратному ходу решения методом Гаусса.

В последнем уравнении системы получилось: , и теперь мы легко найдём решение в предпоследнем уравнении: , а из первого уравнения получаем:

= =

В итоге, получился результат, который можно и записать.

Ответ

,

,

,

,

,

.

Примеры решения методом Гаусса

Выше мы подробно расписали решение системы методом Гаусса. Чтобы закрепить материал, решим несколько примеров, в которых опять нам поможет метод Гаусса. Соответственно, начнём с самой простой системы.

Пример 1

Задача 

Решить систему линейных алгебраических уравнений методом Гаусса:

Решение

Выписываем матрицу, куда добавляем столбец свободных членов:

Прежде всего мы смотрим на элемент, который находится в матрице в левом верхнем углу (первая строка, первый столбец). Для наглядности выделим цифру зелёным квадратом. На этом месте практически всегда стоит единица:

Так как мы должны использовать подходящее элементарное преобразование строк и сделать так, чтобы элемент, который находится в матрице под выделенной цифрой превратился в . Для этого можно ко второй строке прибавить первую строку и умножить на .Однако, не сильно хочется работать с дробями, поэтому давайте постараемся этого избежать. Для этого нужно вторую строку умножить на (разрешающий элемент данного шага).

Соответственно, первая строка остаётся неизменной, а вторая поменяется:

Подбираем такое элементарное преобразование строк, чтобы во второй строке в первом столбце образовался . Для этого первую строку нужно умножить на и только после этого ко второй строке прибавить изменённую после умножения на вторую строку. Вот что получилось:

. Теперь прибавляем со второй строки первую строку . У нас получился , который записываем во вторую строку в первый столбец. Также решаем и остальные элементы матрицы. Вот что у нас получилось:

Как всегда у нас первая строка осталась без изменений, а вторая с новыми числами.

Итак, у нас получился ступенчатый вид матрицы:

Записываем новую систему уравнений:

Для проверки решаем систему обратным ходом. Для этого находим сначала :

Так как найден, находим :

.

Подставляем в изначальную нашу систему уравнений найденные и :

и .

Как видите из решения, система уравнений решена верно. Запишем ответ.

Ответ

Выше мы решали систему уравнений в двумя неизвестными, а теперь рассмотрим систему уравнений с тремя неизвестными.

Пример 2

Задача

Решить систему уравнений методом Гаусса:

Решение

Составляем матрицу, куда вписываем и свободные члены:

Что нам надо? Чтобы вместо цифры 2 появился 0. Для этого подбираем ближайшее число. Например, можно взять цифру -2 и на неё перемножить все элементы первой строки. Значит, умножаем , а потом прибавляем, при этом задействуем вторую строку: . В итоге у нас получился нуль, который записываем во вторую строку в первый столбец. Затем , и . Аналогично, и . И умножаем свободный член . Так и запишем следующую матрицу. Не забывайте, что первая строка остаётся без изменений:

Дальше необходимо проделать те же самые действия по отношению к третьей строке. То есть, первую строку нужно умножать не на (-2), а на цифру 3, так как и в третьей строке нужно коэффициенты привести у нулю. Также первую строку умножаем на 3 и прибавляем третью строку. Получается так:

Теперь нужно обнулить элемент 7, который стоит в третьей строке во втором столбце. Для этого выбираем цифру (-7) и проделываем те же действия. Однако, необходимо задействовать вторую строку. То есть, вторую строку умножаем на (-7) и прибавляем с третьей строкой. Итак, . Записываем результат в третью строку. Такие же действия проделываем и с остальными элементами. Получается новая матрица:

В результате получилась ступенчатая система уравнений:

Сначала находим : ,

.

Обратный ход:

Итак, уравнение системы решено верно.

Ответ

,

,

.

Пример 3

Система с четырьмя неизвестными более сложная, так как в ней легко запутаться. Попробуем решить такую систему уравнений.

Задача

Решите систему уравнений методом Гаусса:

Решение                                                                

В уравнении , то есть – ведущий член и пусть  ≠ 0

Из данного уравнения составим расширенную матрицу:

Теперь нужно умножить последние три строки (вторую, третью и четвёртую) на: , , . Затем прибавим полученный результат ко второй, третьей и четвёртой строкам исключаем переменную из каждой строки, начиная не с первой, а не со второй. Посмотрите, как изменилась наша новая матрица и в теперь стоит 0.

Поменяем вторую и третью строку местами и получим:

Получилось так, что = b и тогда, умножая вторую строку на (-7/4) и результат данной строки, прибавляя к четвёртой, можно исключить переменную из третьей и четвёртой строк:

Получилась такая матрица:

Также, учитывая, что  = , умножим третью строку на: 13,5/8 = 27/16, и, полученный результат прибавим к четвёртой, чтобы исключить переменную и получаем новую систему уравнений:

Теперь необходимо решить уравнение обратным ходом и найдём из последнего, четвёртого уравнения ,

из третьего: = = =

второе уравнение находим: = = = 2,

из первого уравнения: = .

Значит, решение системы такое: (1, 2, -1, -2).

Ответ

,

,

,

.

Добавим ещё несколько примеров для закрепления материла, но без такого подробного описания, как предыдущие системы уравнений.

Пример 4

Задача

Решить систему уравнений методом Гаусса:

Решение

Записываем расширенную матрицу системы:

Сначала смотрим на левое верхнее число:

Как выше уже было сказано, на этом месте должна стоять единица, но не обязательно. Производим такие действия: первую строку умножаем на -3, а потом ко второй строке прибавляем первую:

 

Производим следующие действия: первую строку умножаем на -1. Затем к третьей строки прибавляем вторую:

Теперь вторую строку умножаем на 1, а затем к третьей строке прибавляем вторую:

Получился ступенчатый вид уравнения:

Проверяем:

,

,

,

,

.

.

  Ответ

,

,

.

Заключение

Итак, вы видите, что метод Гаусса – интересный и простой способ решения систем линейных алгебраических уравнений. Путём элементарных преобразований нужно из системы исключать неизвестные переменные, чтобы систему превратить в ступенчатый вид. Данный метод удобен тем, что всегда можно проверить, правильно ли решено уравнение. Нужно просто подставить найденные неизвестные в изначальную систему уравнений.

Если элементы определителя не равняются нулю, тогда лучше обратиться к методу Крамера, а если же элементы нулевые, тогда такие системы очень удобно решать благодаря методу Гаусса.

Предлагаем ещё почитать учебники, в которых также описаны решения систем методом Гаусса.

Литература для общего развития:

Умнов А. Е. Аналитическая геометрия и линейная алгебра, изд. 3: учеб. пособие – М. МФТИ – 2011 – 259 с.

Карчевский Е. М. Лекции по линейной алгебре и аналитической геометрии, учеб. пособие – Казанский университет – 2012 – 302 с.

Метод Гаусса – теорема, примеры решений обновлено: 16 апреля, 2020 автором: Научные Статьи.Ру

Решение уравнений методом Гаусса | matematicus.ru

С помощью метода Гаусса можно решить любую систему линейных уравнений с различным числом уравнений и неизвестных переменных. И именно этим свойством этот метод превосходит матричный метод и метод Крамера.

Суть метода состоит в приведении системы линейных уравнений к ступенчатому (треугольному) виду за счет последовательного исключения неизвестных. Затем её решения с помощью обратной подстановки.


Допустимые преобразования матрицы:

  1. Перестановка местами двух строк или двух столбцов;
  2. Умножение строки на число, которое не равно 0;
  3. Прибавление одной строки к другой.
  4. Исключение или добавление нулевой строки

Допустим, дана система линейных алгебраических уравнений с четырьмя уравнениями и четырьмя неизвестными.

Составим расширенную матрицу СЛАУ:

Затем первое уравнение СЛАУ делим на a11.  При этом a11≠0, если равно нуля, то переставляем две строки или два столбца местами так, чтобы избавится от нуля. После полученное уравнение умножаем на a21 и вычитаем из второго уравнения, дальше, умножаем на a31 и вычитаем из третьего уравнения и т.д.

Также поступаем и с оставшемся уравнениями, т.е. со вторым, третьем и четвёртым. В итоге должна получится матрица ступенчатого или треугольного вида.

Система уравнений примет вид

Такую систему элементарно решить обратным ходом, т.е. последовательным решением уравнений от нижнего к верхнему.

Рассмотрим наиболее подробно метод Гаусса при решении СЛАУ на практике.

Пример 1

Решить методом Гаусса систему уравнений

Решение

Составим расширенную матрицу системы уравнений:

Первую строку разделим на a11, но так как в этой строке a11=0, то необходимо поменять строку у которой первый элемент не равен нулю. Выберем по модулю наибольшей элемент, это a41=2 Поэтому поменяем первую и четвёртую строки местами.

Получаем:

Первую строку разделим на a11=2. Получим матрицу:

Умножаем элементы первой строки на -1 и прибавляем к элементам второй строк. Получим матрицу:

Умножаем элементы первой строки на -1 и прибавляем к элементам третьей строки.

Четвёртую строку оставляем без изменений, так как её первый элемент равен нулю.

Теперь первый столбец не трогаем.

Начинаем повторять действия, которые применяли ранее.

Второе уравнение разделим на a22=-1/2, тогда

Умножаем элементы второй строки на -1/2 и прибавляем к элементам третьей строки.

Умножаем элементы второй строки на -1 и прибавляем к элементам четвёртой строки.

Первый и второй столбец не трогаем.

Третьей столбец разделим на 2.

Умножаем элементы третьей строки на -1 и прибавляем к элементам четвёртой строки.

Получаем ступенчатую систему алгебраических уравнений:

Отсюда, решая систему снизу вверх, получаем корни системы уравнений


Приведём простой пример краткой записи решения СЛАУ методом Гаусса

Пример 2

Решить систему линейных уравнений с тремя неизвестными методом Гаусса.

Решение

Составим расширенную матрицу системы линейных уравнений .

Следовательно, искомая система может быть представлена в ступенчатом виде:

Решая последовательно уравнение, получаем:

Ответ: z = 3; y = 2; x = 1

Метод Гаусса. Примеры

Метод Гаусса заключается в последовательном исключении переменных и преобразовании системы линейных алгебраических уравнений

к треугольному виду

Предположим, что в системе коэффициент . Если это условие не выполняется, то на первое место переносим уравнение, которое ее удовлетворяет. С помощью первого уравнения исключим из остальных уравнений.

Для этого делят первую строчку на , обозначим

.

Дальше второй строки вычитаем первую строку, умноженную на ;от третьего первую строчку, умноженный на ; и так далее до последней строки. Получим таблицу коэффициентов:

Для неизвестных имеем систему уравнений. Выполняя, как и раньше, исключим из всех уравнений, начиная с третьего. Для этого сначала разделим вторую строчку на .

Если коэффициент , то переставим уравнения так, чтобы выполнялось условие .

Обозначив

,

от третьей строки вычтем вторую строчку, умноженный на ;

от четвертой строки вычтем вторую строчку, умноженный на и т.д. Получим таблицу коэффициентов:

Продолжая процесс исключения неизвестных получим таблицу:

Таблица коэффициентов при неизвестных сводится к треугольному виду. Все главной диагонали элементы . Запишем соответствующую систему уравнений:

Переход от первой системы уравнений до последней называется прямым ходом метода Гаусса. Обратный ход метода Гаусса начинается с последней системы уравнений. Ее решают с конца до начала. Из последнего уравнения находят . Подставив это значение в предпоследнее — находят и т.д. Из первого уравнения находят .

Если система уравнений с неизвестными имеет единственное решение, то эта система всегда может быть преобразована к треугольному виду. Для студентов не всегда требуют, чтобы диагональные элементы были равны единице. Достаточно просто свести систему линейных уравнений к верхней треугольной.

———————————————

Пример 1.

Дана система трех линейных уравнений с тремя неизвестными. Решить систему методом Гаусса.

Решение.

Исключим неизвестную из второго и третьего уравнения. Для этого от них вычтем первое умноженное на

Видим, что наше уравнение в таком виде можно решать обратным ходом метода Гаусса. Для этого из последнего уравнения выразим

Подставим полученное значение в предыдущее уравнение и найдем

Из первого уравнения находим

Решение данной системы равен

——————————————

В случаях систем больших размеров, а также для удобства, часто на практике используют другую схему решения. Вместо преобразований над системой выполняют соответствующие преобразования над матрицей, составленной из коэффициентов при неизвестных и столбца из свободных членов, который для удобства выделяют вертикальной линией. Такую матрицу называют расширенной матрицей системы.

——————————————

Пример 2.

Решить систему четырех линейных алгебраических уравнений методом Гаусса.

Решение.

Выпишем расширенную матрицу для данной системы

Сведем ее к треугольному виду с помощью элементарных преобразований.

1.Поменяем местами первый и второй строки.

2. Добавим к элементам второго, третьего и четвертого строк элементы первой строки, умноженные соответственно на

3. Поменяем местами второй и третий строки. Добавим к элементам третьего и четвертого строк элементы второй строки, умноженные соответственно на

4. От четвертого уравнения умноженного на вычитаем третье уравнение умноженное на

Такой расширенной матрицы соответствует следующая система уравнений

С четвертого уравнения находим и подставляем в третье уравнение

Найденные значения подставляем во второе уравнение

Из первого уравнения находим первую неизвестную

Система полностью решена и – ее решение.

——————————————————

Посмотреть материалы:

в чем суть, решение системы уравнений, примеры с объяснением

Благодаря великим ученым было открыто множество эффективных теорем для работы со сложными математическими задачами. Один из таких примеров — метод Гаусса.

Метод Гаусса — что это такое

Метод Гаусса представляет собой методику эквивалентного преобразования исходной системы линейных уравнений в систему, решаемую существенно проще, чем исходный вариант.

Метод Гаусса используют для решения систем линейных алгебраических формул. Такой способ обладает рядом важных преимуществ:

  1. Нет необходимости сравнивать уравнения для оценки совместимости.
  2. Решение систем равенств, в которых число определителей совпадает или не совпадает с количеством неизвестных переменных.
  3. Поиск решений для уравнений с нулевым определителем.
  4. Сравнительно небольшое количество вычислительных операций для получения результата.

Основные определения и обозначения

Матрицы: определение и свойства

Такие системы являются наиболее удобным способом представления данных, с которыми впоследствии производят манипуляции. Матрица имеет вид прямоугольника для удобства расчетов. При использовании метода Гаусса работа осуществляется с треугольными матрицами, при записи которых применяется прямоугольник с нулями на тех местах, где числа отсутствуют. Часто нули не записывают, а только подразумевают.

Важным параметром матрицы является размер:

  • ширина — это количество строк, обозначают буквой m;
  • длину выражают числом столбцов, записывают буквой n.
Источник: bigpicture.ru

Размер матрицы будет записан в формате А m*n. В случае, когда m=n, матрица является квадратной, а m=n служит ее порядком. Номера строк и столбцов изменяются.

Определитель

Матрица обладает крайне важной характеристикой. Таким параметром является определитель. Данную величину рассчитывают с помощью диагонали. Для этого в матрице необходимо провести воображаемые диагональные линии. Затем следует найти произведение элементов, которые располагаются на этих диагоналях, а полученные значения суммировать таким образом:

  1. Если диагональ обладает наклоном в правую сторону, то знак «+».
  2. Для диагоналей, наклоненных влево, знак «–».
Источник: wp.com

Рассчитать определитель представляется возможным лишь в случае работы с квадратной матрицей.

Если необходимо определить данный параметр для прямоугольной матрицы, то следует выполнить следующие манипуляции:

  • из числа строк и числа столбцов выбрать наименьшее и обозначить его k;
  • отметить в матрице произвольным образом k столбцов и k строк.

Элементы, которые расположены на пересечении отмеченных столбцов и строк, образуют новую квадратную матрицу. В случае, когда определитель является числом, не равным нулю, то данный параметр будет обозначен как базисный минор первоначальной прямоугольной матрицы. Перед решением систем уравнений методом Гаусса полезно рассчитать определитель. Если данная характеристика равна нулю, то матрица имеет бесконечное множество решений либо не имеет их вовсе. В таком случае потребуется определить ранг матрицы.

Классификация систем

Ранг матрицы является распространенным понятием. Он обозначает максимальный порядок ее определителя, который не равен нулю. По-другому можно сказать, что ранг матрицы представляет собой порядок базисного минора. Исходя из данного критерия, СЛАУ классифицируют на несколько типов. В совместных системах, которые состоят лишь из коэффициентов, ранг основной матрицы совпадает с рангом расширенной. Для подобных систем характерно одно или множество решений. По этой причине совместные системы подразделяют на следующие типы:

  • определенные, обладающие одним решением, в которых наблюдается равенство ранга матрицы и количество неизвестных;
  • неопределенные;
  • обладающие бесконечным числом решений с рангом матрицы, который меньше количества неизвестных.

В несовместных системах ранги, характеризующие основную и расширенную матрицы, отличаются. С помощью метода Гаусса в процессе решения можно прийти либо к однозначному доказательству несовместности системы, либо к решению общего вида для системы, обладающей бесконечным количеством решений.

Источник: asiaplustj.info

Основные правила и разрешаемые преобразования при использовании метода Гаусса

Перед тем, как решать систему, необходимо ее упростить. На данном этапе выполняют элементарные преобразования, которые не влияют на конечный результат. Определенные манипуляции справедливы лишь в случае матриц, исходниками которых являются СЛАУ. Список элементарных преобразований:

  1. Перестановка строк. При перемене записей в системе местами ее решение не меняется. Можно менять место строк в матрице, учитывая столбец со свободными членами.
  2. Произведение всех элементов строк и некоторого коэффициента. Сокращаются большие числа в матрице, и исключаются нули. При этом множество решений сохраняется без изменений, а дальнейшие манипуляции существенно упрощаются. Важным условием является отличие от нуля коэффициента.
  3. Удаление строк, которые содержат пропорциональные коэффициенты. Данное преобразование следует из предыдущего пункта. При условии, что две или более строк в матрице обладают пропорциональными коэффициентами, то при произведении или делении одной из строк на коэффициент пропорциональности получают две или более абсолютно одинаковые строки. В этом случае лишние строки исключают, оставляя только одну.
  4. Удаление нулевой строки. Бывают случаи, когда в процессе манипуляций с уравнениями возникает строка, все элементы которой, в том числе свободный член, равны нулю. Нулевую строку допустимо исключать из матрицы.
  5. Суммирование элементов одной строки с элементами другой, умноженными на некоторый коэффициент, в соответствующих столбцах. Данное преобразование имеет наиболее важное значение из всех перечисленных.

Особенности использования метода Гаусса для решения СЛАУ

На первом этапе система уравнений записывается в определенном виде. Пример выглядит следующим образом:

Источник: wp.com

Коэффициенты необходимо представить в виде таблицы. С правой стороны в отдельном столбце записаны свободные члены. Данный блок отделен для удобства решения. Матрицу со столбцом со свободными членами называют расширенной.

Источник: wp.com

Затем основная матрица с коэффициентами приводится к верхней треугольной форме. Данное действие является ключевым моментом при решении системы уравнений с помощью метода Гаусса. По итогам преобразований матрица должна приобрести такой вид, чтобы слева внизу находились одни нули:

Источник: wp.com

При записи новой матрицы в виде системы уравнений можно отметить, что последняя строка уже содержит значение одного из корней, которое в дальнейшем подставляется в уравнение выше для нахождения следующего корня и так далее. Подобное описание позволяет разобраться в методе Гаусса в общих чертах.

Обратный и прямой ход метода Гаусса

В первом случае необходимо представить запись расширенной матрицы системы. При выполнении обратного метода Гаусса далее в главную матрицу добавляют столбец со свободными членами.

Источник: wp.com

Суть такого способа заключается в выполнении элементарных преобразований, по итогам которых данная матрица приводится к ступенчатому или треугольному виду. В этом случае над или под главной диагональю матрицы располагаются только нули.

Источник: wp.com

Варианты дальнейших действий:

  • перемена строк матрицы местами, при наличии одинаковых или пропорциональных строк их можно исключить, кроме одной;
  • деление либо умножение строки на любое число, не равное нулю;
  • удаление нулевых строк;
  • добавление строки, умноженной на число, не равное нулю, к другой строке.

Имея преобразованную систему с одной неизвестной Xn, которая становится известной, можно выполнить поиск в обратном порядке остальных неизвестных с помощью подстановки известных х в уравнения системы, вплоть до первого. Данный способ называют обратным методом Гаусса.

Примеры решений с объяснением

Пример 1

Требуется решить с помощью метода Гаусса систему линейных уравнений, которая выглядит следующим образом:

Источник: wp. com

Решение

Необходимо записать расширенную матрицу:

Источник: wp.com

Затем нужно выполнить преобразования. В результате матрица должна приобрести треугольный вид. Для этого следует умножить первую строку на (3) и умножить вторую строку на (-1). В результате суммирования второй и первой строк получается следующее:

Источник: wp.com

Далее следует умножить третью строку на (-1). После добавления третьей строки ко второй получаем следующие преобразования:

Источник: wp.com

После этого необходимо умножить первую строку на (6) и вторую строку на (13). Далее следует добавить вторую строку к первой:

Источник: wp.com

После того, как система преобразована, остается вычислить неизвестные:

\(x_{3}=\frac{98}{49}=2\)

\(x_{2}=\frac{14-7x_{3}}{6}=\frac{14-7*2}{6}=0\)

\(x_{3}=\frac{-9+5x_{2}+6x_{3}}{3}=\frac{-9+5*0+6*2}{3}=1\)

Данный пример демонстрирует единственное решение системы.

Источник: supertics.com

Пример 2

Необходимо решить систему уравнений, которая выглядит следующим образом:

Источник: wp.com

Решение

Необходимо составить матрицу:

Источник: wp.com

Согласно методу Гаусса уравнение первой строки по итогам преобразований не меняется. Удобнее, когда левый верхний элемент матрицы обладает наименьшим значением. В таком случае первые элементы остальных строк после преобразований будут равны нулю. Таким образом, составленная матрица будет решаться проще, если на место первой строки поставить вторую:

вторая строка:

\(k = (-a_{21} /a_{11}) = (-3/1) = -3\)

\(a»_{21} = a_{21} + k×a_{11} = 3 + (-3)×1 = 0\)

\(a» _{22} = a_{22} + k×a _{12} = -1 + (-3)×2 = -7\)

\(a»_{ 23} = a_{23} + k×a_{13} = 1 + (-3)×4 = -11\)

b» 2 = b 2 + k×b 1 = 12 + (-3)×12 = -24

третья строка: 

\(k = (-a_{31} /a_{11}) = (-5/1) = -5\)

\(a»_{31} = a_{31} + k×a_{11} = 5 + (-5)×1 = 0\)

\(a»_{32} = a_{32} + k×a_{12} = 1 + (-5)×2 = -9\)

\( a»_{33} = a_{33} + k×a_{13} = 2 + (-5)×4 = -18\)

\( b»_3 = b_3 + k×b_1 = 3 + (-5)×12 = -57\)

Матрица с промежуточными результатами манипуляций будет иметь следующий вид:

Источник: wp.com

Благодаря некоторым операциям можно придать матрице наиболее удобный вид. К примеру, вторую строку можно избавить от всех «минусов» путем умножения каждого элемента на «-1». Можно заметить, что для третьей строки характерны все элементы, кратные трем. В этом случае строка сокращается с помощью произведения каждого элемента на «-1/3». Минус позволит удалить отрицательные значения.

Источник: wp.com

Далее следует приступить к манипуляциям со второй и третьей строками. Необходимо суммировать третью и вторую строки. Вторая строка при этом умножается на такой коэффициент, при котором элемент а 32 будет равен нулю.

\(k = (-a_{32} /a_{22}) = (-3/7) = -3/7\)

В случае, когда некоторые преобразования приводят в результате к получению не целого числа, следует оставить его в этом виде. Таким образом, вычисления будут более точными. Затем при получении ответов можно определиться с его дальнейшем округлением или переводом в другую форму записи.

\(a»_{32} = a_{32} + k×a_{22} = 3 + (-3/7)×7 = 3 + (-3) = 0\)

\(a»_{33} = a_{33} + k×a_{23} = 6 + (-3/7)×11 = -9/7\)

\(b»_3 = b_3 + k×b_2 = 19 + (-3/7)×24 = -61/7\)

Преобразованная матрица будет иметь следующий вид:

 

 

Матрица обладает ступенчатым видом. Дальнейшие преобразования с помощью метода Гаусса нецелесообразны. В этом случае можно удалить из третьей строки общий коэффициент «-1/7».

Источник: wp.com

Затем необходимо представить запись матрицы в виде системы уравнений для вычисления корней.

x + 2y + 4z = 12 (1)

7y + 11z = 24 (2)

Найти корни можно обратным методом Гаусса. Уравнение (3) содержит значение z:

y = (24 — 11×(61/9))/7 = -65/9

С помощью первого уравнения можно определить х:

x = (12 — 4z — 2y)/1 = 12 — 4×(61/9) — 2×(-65/9) = -6/9 = -2/3

Подобная система является совместной и определенной, для которого характерно единственное решение. Ответ будет следующим:

x 1 = -2/3, y = -65/9, z = 61/9.

Метод Гаусса предполагает последовательное исключение неизвестных. Методика справедлива в случае решения квадратных систем линейных алгебраических уравнений. Несмотря на простоту метода, многие студенты сталкиваются с некоторыми трудностями в процессе поиска правильного решения. Это связано с наличием знаков «+» и «-». Поэтому для решения СЛАУ требуется проявить внимательность. А получить квалифицированную помощь можно на ресурсе Феникс.Хелп.

Решение системы линейных алгебраических уравнений методом Гаусса

Система линейных уравнений вида:

может быть решена методом Гаусса при помощи нашего калькулятора.

Система уравнений задается в виде расширенной матрицы, т. е. матрицы коэффициентов и свободных членов размерности [n : n+1] вида:

Описание метода Гаусса следует сразу за калькулятором.

Решение системы линейных уравнений методом Гаусса
8 3 4 5 31 14 4 33 23 17 15 4 23 7 22 4 11 17 1 51

СЛАУ в матричном виде

Точность вычисления

Знаков после запятой: 2

Количество решений

 

Вектор решения системы уравнений

 

Файл очень большой, при загрузке и создании может наблюдаться торможение браузера.

Загрузить close

content_copy Ссылка save Сохранить extension Виджет

Метод Гаусса

Метод был назван в честь гениального немецкого математика XIX века Карла Фридриха Гаусса. Сам Гаусс не был первооткрывателем метода (метод был известен и ранее (еще в I-II веке до н. э. метод упоминался в китайском труде «Математика в девяти книгах»).

Приведение матрицы к ступенчатому виду

На первом шаге решения системы уравнений методом Гаусса матрица коэффициентов и свободных членов приводится к ступенчатому виду:

Матрица превращается в ступенчатую форму путем элементарных преобразований — перемена строк местами, умножение строки на коэффициент, сложение строк.
В нашем калькуляторе для перехода к ступенчатому виду осуществляется последовательное вычитание из нижних строк матрицы, помноженных на , верхних строк , помноженных на коэффициент , где i — индекс текущей строки (индекс строки, которую вычитают из нижних строк).
При осуществлении этой операции требуется, чтобы коэффициент главной переменной был не нулевым. В случае нулевого коэффициента, строка меняется местами с любой другой нижней строкой, в которой в текущем столбце значение отлично от нуля.

Выражение базисных переменных

Получив ступенчатую матрицу, мы переходим к выражению базисных переменных, для этого сначала выполняется деление текущей строки на коэффициент , затем производится обратное вычитание из верхних строк , этой строки , помноженных на коэффициент , где j — индекс текущей строки (индекс строки, которую вычитают из верхних строк). Операция повторяется с каждой строкой, начиная от n-й до 1-й.
В результате матрица приобретает диагональный вид:
,
далее, поделив строки матрицы на коэффициент , в столбце свободных членов получаем вектор решений системы уравнений.

Решение систем уравнений методом Гаусса | Проект по алгебре (8 класс) на тему:

МАОУ-лицей № 13 п. Краснообск, Рязанова М. «Решение систем уравнений методом Гаусса»

Муниципальное автономное общеобразовательное учреждение

        Новосибирского района Новосибирской области –

      лицей № 13 п. Краснообск

Решение систем уравнений

методом Гаусса

Работу выполнила:

Рязанова Мария

8Б класс

Руководитель:

учитель математики

Черемисина Галина Артуровна

                                                                                                                           

   

2018

Содержание:

  1. Введение…….…………………………………..…………….…..3
  2.      Немного из биографии Гаусса……………………………………4
  3. Понятие матрицы и  её преобразования ……………………….…5
  4. Решение двойной системы уравнений………………….……….7
  5. Решение тройной системы……………………………………….8
  6.  Заключение ………….…………………………….……………..9
  7. Используемые ресурсы……………………………………….…10

Введение

Одной из основных задач алгебры является решение систем линейных алгебраических уравнений. Большая  часть методов решения различных задач включает в себя решение систем линейных уравнений как один из шагов соответствующего алгоритма. Достаточно известным методом решения систем линейных уравнений является метод Гаусса. Это  метод также называют методом последовательного исключения неизвестных Гаусса, который является одним из наиболее универсальных и эффективных методов решения линейных систем уравнений,  известный в различных вариантах уже более 2000 лет.

Процесс решения по методу Гаусса состоит из двух этапов, называемых прямым и обратным ходом. На первом этапе система приводится к треугольному виду, а на втором (обратный ход) идет последовательное определение неизвестных из указанной треугольной системы.

Метод Гаусса — один из основных результатов линейной алгебры и аналитической геометрии, к нему сводятся множество других теорем и методов линейной алгебры. Поэтому поиск решения системы линейных уравнений методом Гаусса имеет не только важное значение, но и является частью алгоритма решения многих задач, что позволяет говорить об актуальности изучения метода Гаусса. В отличие от матричного метода и метода Крамера, метод Гаусса может быть применен к системам линейных уравнений с произвольным числом уравнений и неизвестных. 

Цели и задачи

Цели проекта: ознакомить и научить одноклассников решать системы уравнений методом Гаусса.

Задачи проекта:

  • подобрать информацию по данной теме;
  • изучить метод Гаусса;
  • научиться самостоятельно решать системы уравнений методом Гаусса;
  • рассказать историю появления метода Гаусса, о самом Гауссе и его научных трудах;
  • показать преобразования  матрицы одноклассникам;
  • подобрать и решить примеры систем уравнений этим методом;
  • применить метод Гаусса при решении систем уравнений вместе с одноклассниками.

Немного из биографии Гаусса

Иога́нн Карл Фри́дрих Га́усс (1777 — 1855) — немецкий математик, механик, физик, астроном и геодезист. Считается одним из величайших математиков всех времён, «королём математиков».

С именем Гаусса связаны фундаментальные исследования почти во всех основных областях математики: в алгебре, теории чисел, дифференциальной и неевклидовой геометрии,  теории вероятностей, а также в механике, астрономии, физике и геодезии.

Гаусс чрезвычайно строго относился к своим печатным трудам и никогда не публиковал даже выдающиеся результаты, если считал свою работу над этой темой незавершённой. Изучение архива Гаусса показало, что он медлил с публикацией ряда своих открытий, и в результате его опередили другие математики. Вот неполный перечень упущенных им трудов.

  • Неевклидова геометрия
  • Эллиптические функции
  • Метод наименьших квадратов
  • Закон распределения простых чисел

Метод Гаусса

Метод Гаусса — классический метод решения системы линейных алгебраических уравнений (СЛАУ). Назван в честь немецкого математика Карла Фридриха Гаусса. Это метод последовательного исключения переменных, при котором с помощью элементарных преобразований система уравнений приводится к равносильной системе треугольного вида.  Затем последовательно, начиная с последних (по номеру), находятся все переменные системы. Хотя в настоящее время данный метод повсеместно называется методом Гаусса, он был известен и до К. Ф. Гаусса. Первое известное описание данного метода — в китайском трактате «Математика в девяти книгах».

Понятие матрицы, её преобразования

Матрица (математика) — прямоугольная таблица элементов.

         пример матрицы  

Со строками матрицы мы можем выполнять следующие операции: деление; умножение на число, отличное от нуля; сложение; вычитание.

Существуют следующие элементарные преобразования: 

1) Строки матрицы можно переставлять местами. Например, в рассматриваемой матрице можно безболезненно переставить первую и вторую строки: 

            

2) Если в матрице есть пропорциональные (или одинаковые) строки, то следует удалить из матрицы все эти строки кроме одной. Рассмотрим, например матрицу.

В данной матрице последние три строки пропорциональны, поэтому достаточно оставить только одну из них:  

3) Если в матрице в ходе преобразований появилась нулевая строка, то ее также следует удалить. Нулевая строка – это строка, в которой одни нули.

4) Строку матрицы можно умножить (разделить) на любое число, отличное от нуля. Рассмотрим, например, матрицу: 

Здесь целесообразно первую строку разделить на –3, а вторую строку – умножить на 2:

                                   

Данное действие очень полезно, поскольку упрощает дальнейшие преобразования матрицы.

5) К строке матрицы можно прибавить другую строку, умноженную на число, отличное от нуля.

Строка, которую ПРИБАВЛЯЛИ – не изменилась. Всегда меняется строка, К КОТОРОЙ  ПРИБАВЛЯЮТ.

Цель элементарных преобразований – привести матрицу к ступенчатому виду: 

Сам термин «ступенчатый вид» не вполне теоретический, в научной и учебной литературе он часто называется трапециевидный вид или треугольный вид. 

Решение системы уравнений с двумя переменными    

      Решим данную систему методом Гаусса.

На первом этапе нужно записать расширенную матрицу системы,

которая представляет из себя коэффициенты чисел:

  1. Умножаем первую строку на 2
  2. К первой строке прибавляем вторую
  3. Сокращаем первую строку на 5
  4. Умножаем первую строку на 3
  5. Из второй строки вычитаем первую
  6. Первую строку сокращаем на 3, а вторую на 4

                                                                                        

                                                                                                        

            

    Ответ:

Решение системы уравнений с тремя переменными    

  1. Вычитаем из первой строки вторую
  2. Вычитаем из первой строки первую строку, умноженную на 2. Из третьей строки вычитаем первую.

             

  1. Третью строку умножаем на -1.
  2. Ко второй строке прибавляем третью, умноженную на 7.

  1. Вторую строку делим на -30.
  2. Из первой строки вычитаем третью, умноженную на 3.

  1. Из первой строки вычитаем вторую, умноженную на 13.
  2. К третьей строке прибавляем вторую, умноженную на 7. 

                        — решение системы уравнений

       

    Ответ:

Заключение

Поработав с данным методом, я ощутила преимущество его применения по отношению к классическим приёмам решения систем уравнений. Надеюсь, знание этого метода мне поможет в будущем быстро решать системы уравнений с несколькими переменными, а также осуществлять проверку решений классическими способами.

Одной из целей моего проекта является научить одноклассников решать системы уравнений методом Гаусса, для чего на спецкурсе по математике я показала и рассказала им о своей работе и предложила совместно решить несколько систем, состоящих из двух строк, методом Гаусса. Ребят заинтересовал данный метод, они с интересом слушали меня, а потом совместно решали системы уравнений данным методом.  Далее предложены системы уравнений и фотографии с нашего занятия.

   Ответ:     Ответ:  

    Ответ:          Ответ:  

 Ответ:             Ответ:

   Ответ:

 

У меня получилось самой освоить данный метод и передать свои знания одноклассникам. Таким образом,  поставленные мною цели и задачи выполнены. И ещё метод Гаусса прост тем, как мне кажется, что для его освоения не требуется много знаний. А также для матриц ограниченного размера метод Гаусса менее трудоёмкий по сравнению с другими методами, поэтому в будущем я планирую его активно применять при решении систем уравнений.

Используемые ресурсы:

Методические рекомендации к внеурочной самостоятельной работе «Решение систем уравнений методом Гаусса»

Самостоятельная работа №2

Решение систем уравнений методом Гаусса

Цель работы: овладеть методом Гаусса при решении систем линейных уравнений

Студент должен:

Знать:

  • символику и формы записи систем линейных уравнений

  • что такое совместная и несовместная система уравнений

  • методы решения СЛАУ(метод Гаусса)

Уметь:

КОНТРОЛЬНЫЕ ВОПРОСЫ:

  1. В каком случае система имеет единственное решение?

  2. В каком случае система имеет бесконечное множество решений?

  3. В чем достоинство метода Гаусса по сравнению с другими методами?

Форма выполнения задания: решение задач (письменно)

Время выполнения 45 мин

Основной теоретический материал

Метод Гаусса. Этот способ заключается в обнулении элементов основной расширенной матрицы системы уравнений, находящихся под главной диагональю.

Ме́тод Га́усса — классический метод решения системы линейных алгебраических уравнений (СЛАУ). Это метод последовательного исключения переменных, когда с помощью элементарных преобразований система уравнений приводится к равносильной системе треугольного вида, из которой последовательно, начиная с последних (по номеру), находятся все переменные системы

Алгоритм метода Гаусса

  1. На основании системы линейных уравнений составляем расширенную матрицу системы;

  2. Приводим матрицу к «треугольному» виду;

  3. Определяем ранги основной и расширенной матриц, и на основании этого делаем вывод о совместности системы и количестве допустимых решений;

  4. В случае, если система имеет единственное решение производим обратную подстановку и находим его, если система имеет множество решений: выражаем базисные переменные через переменные которые могут принимать произвольные значения;

Для приведения исходной расширенной матрицы к треугольному виду используем следующие два свойства определителей:

Свойство 1. Определитель не изменит свое значение, если ко всем элементам какой-либо строки (столбца) матрицы прибавить соответствующие элементы параллельной строки (столбца), умноженные на произвольное одно и то же число.

Свойство 2. При перестановке двух любых столбцов или строк матрицы ее определитель меняет знак на противоположный, а абсолютная величина определителя остается неизменной.

На первом этапе составляется расширенная матрица, состоящая из коэффициентов при неизвестных, и с помощью несложных математических преобразований она приводится к виду, когда диагональ, состоящая из единичек отсекает нули:

  

На втором этапе последовательно находятся все неизвестные, начиная с предпоследней.

Решение типовых заданий

Решить систему трех линейных уравнений методом Гаусса

Ответ: х=1, y=2, z=3.

Покажем, как методом Гаусса можно решить следующую систему:

Обнулим коэффициенты при во второй и третьей строчках. Для этого вычтем из них первую строчку, умноженную на и , соответственно:

Теперь обнулим коэффициент при в третьей строке, вычтя из неё вторую строку, умноженную на :

В результате мы привели исходную систему к треугольному виду, тем самым закончим первый этап алгоритма.

На втором этапе разрешим полученные уравнения в обратном порядке. Имеем:

из третьего;

из второго, подставив полученное

из первого, подставив полученные и .

Ответ: (2; 3; -1).

Решить самостоятельно системы линейных уравнений по вариантам: 10 вариантов

1

2

3

4

5

6

7

8

9

10

Оформить отчет

Требования к оформлению самостоятельной работы

Расчетные задания должны быть выполнены в рабочей тетради

По результатам решения тренажера выставляется оценка, которая учитывается при приеме дифференцированного зачета.

Шкала оценки образовательных достижений

Процент результативности

(правильных ответов)

Интернет ресурсы

http://math2.ru/education/sys_lin_eq/gauss.html

http://www.bestreferat.ru/referat-180751.html

Оценка уровня подготовки

Балл (оценка)

Вербальный аналог

90-100

5

отлично

80-89

4

хорошо

70-79

3

удовлетворительно

менее 70

2

неудовлетворительно

Исключение Гаусса

Тип 2. Умножьте строку на ненулевую константу.

Тип 3. Добавьте одну строку, кратную одной, в другую.

Цель этих операций — преобразовать — или уменьшить — исходную расширенную матрицу в одну из форм, где A ′ является верхним треугольником ( a ij ′ = 0 для i> j ), любые нулевые строки появляются внизу матрицы, и первая ненулевая запись в любой строке находится справа от первой ненулевой записи в любой более высокой строке; такая матрица называется эшелон .Решения системы представлены более простой расширенной матрицей [ A ′ | b ′], можно найти путем осмотра нижних рядов и обратной подстановки в более высокие ряды. Поскольку элементарные операции со строками не меняют решений системы, векторы x , которые удовлетворяют более простой системе A x = b ′, являются в точности теми, которые удовлетворяют исходной системе, A x = b .

Пример 3 : Решите следующую систему с помощью исключения Гаусса:

Расширенная матрица, которая представляет эту систему:

Первая цель — получить нули под первой записью в первом столбце , что означает исключение первой переменной x из второго и третьего уравнений.Для этого выполняются следующие операции со строками:

Вторая цель — получить ноль под второй записью во втором столбце, что означает исключение второй переменной y из третьего уравнения. Один из способов добиться этого — добавить -1/5 второй строки к третьей строке. Однако, чтобы избежать дробей, есть еще один вариант: сначала поменять местами второй и третий ряды. Замена двух строк просто меняет местами уравнения, что явно не изменит решения системы:

Теперь прибавьте −5 раз вторую строку к третьей строке:

Поскольку матрица коэффициентов преобразована в эшелонированную форму, «прямая» часть исключения Гаусса завершена.Теперь остается использовать третью строку для оценки третьего неизвестного, затем выполнить обратную подстановку во вторую строку для оценки второго неизвестного и, наконец, выполнить обратную замену в первой строке для оценки первого неизвестного.

Третья строка финальной матрицы переводится в 10 z = 10, что дает z = 1. Обратная подстановка этого значения во вторую строку, которая представляет уравнение y — 3 z = — 1, дает y = 2.Обратная подстановка обоих этих значений в первую строку, которая представляет уравнение x -2 y + z = 0, дает x = 3. Таким образом, решение этой системы: ( x, y, z ) = (3, 2, 1).

Пример 4 : Решите следующую систему с помощью исключения Гаусса:

Для этой системы расширенная матрица (вертикальная линия опущена) составляет

Сначала умножьте строку 1 на 1/2:

Теперь добавление −1 первой строки ко второй строке дает нули под первой записью в первом столбце:

Перестановка второй и третьей строк дает желаемую матрицу коэффициентов верхней треугольной формы:

В третьей строке теперь указано z = 4.Обратная подстановка этого значения во вторую строку дает y = 1, а обратная подстановка обоих этих значений в первую строку дает x = −2. Таким образом, решение этой системы ( x, y, z ) = (−2, 1, 4).

Исключение Гаусса-Джордана . Исключение Гаусса осуществляется путем выполнения элементарных операций со строками для получения нулей ниже диагонали матрицы коэффициентов, чтобы привести ее к эшелонированной форме. (Напомним, что матрица A ′ = [ a ij ′] имеет эшелонированную форму, когда a ij ′ = 0 для i> j , любые нулевые строки появляются в нижней части матрицы , а первая ненулевая запись в любой строке находится справа от первой ненулевой записи в любой более высокой строке.Как только это будет сделано, проверка нижней строки (строк) и обратная подстановка в верхние строки определяют значения неизвестных.

Однако можно сократить (или полностью исключить) вычисления, связанные с обратной подстановкой, выполнив дополнительные операции со строками для преобразования матрицы из эшелонированной формы в сокращенную форму . Матрица находится в форме сокращенного эшелона, когда, помимо того, что она находится в форме эшелона, каждый столбец, содержащий ненулевую запись (обычно равную 1), имеет нули не только под этой записью, но и над этой записью.Грубо говоря, гауссовское исключение работает сверху вниз, чтобы создать матрицу в форме эшелона, тогда как Гаусс-Джордан исключение продолжается с того места, где остановился гауссиан, затем работает снизу вверх для создания матрицы в форме сокращенного эшелона. Техника будет проиллюстрирована на следующем примере.

Пример 5 : Известно, что высота, y , брошенного в воздух объекта задается квадратичной функцией от t (время) в форме y = at 2 + bt + c .Если объект находится на высоте y = 23/4 в момент времени t = 1/2, при y = 7 в момент времени t = 1 и при y = 2 при t = 2 , определите коэффициенты a, b и c .

Так как t = 1/2 дает y = 23/4

, а два других условия, y ( t = 1) = 7 и y ( t = 2) = 2, дают следующие уравнения для a, b и c :

Следовательно, цель — решить систему

Расширенная матрица для этой системы сокращается следующим образом:

На этом прямая часть исключения Гаусса завершена, так как матрица коэффициентов приведена к эшелонированной форме.Однако, чтобы проиллюстрировать исключение Гаусса-Жордана, выполняются следующие дополнительные элементарные операции со строками:

Эта окончательная матрица сразу дает решение: a = −5, b = 10 и c = 2.

Пример 6 : Решите следующую систему с помощью исключения Гаусса:

Расширенная матрица для этой системы —

Кратные значения первой строки добавляются к другим строкам, чтобы получить нули под первой записью в первом столбце:

Затем −1 раз вторая строка добавляется к третьей строке:

В третьей строке теперь указано 0 x + 0 y + 0 z = 1, уравнение, которому не могут соответствовать никакие значения x, y и z .Процесс останавливается: у этой системы нет решений.

Предыдущий пример показывает, как исключение Гаусса выявляет противоречивую систему. Небольшое изменение этой системы (например, изменение постоянного члена «7» в третьем уравнении на «6») проиллюстрирует систему с бесконечным числом решений.

Пример 7 : Решите следующую систему с помощью исключения Гаусса:

Те же операции, которые применяются к расширенной матрице системы в примере 6, применяются к расширенной матрице для данной системы:

Здесь третья строка переводится в 0 x + 0 y + 0 z = 0, уравнение, которому удовлетворяют любые x, y и z .Поскольку здесь нет ограничений на неизвестные, на неизвестные не три условия, а только два (представленные двумя ненулевыми строками в окончательной расширенной матрице). Поскольку есть 3 неизвестных, но только 2 константы, 3–2 = 1 из неизвестных, скажем, z , произвольно; это называется свободной переменной . Пусть z = t , где t — любое действительное число. Обратная подстановка z = t во вторую строку (- y + 5 z = −6) дает

Обратная подстановка z = t и y = 6 + 5 t в первую строку ( x + y -3 z = 4) определяет x :

Следовательно, каждое решение системы имеет вид

, где t — любое действительное число.Существует бесконечно много решений, поскольку каждое действительное значение т дает свое конкретное решение. Например, выбор t = 1 дает ( x, y, z ) = (−4, 11, 1), а t = 3 дает ( x, y, z ) = (4, — 9, −3) и т. Д. Геометрически эта система представляет собой три плоскости в R 3 , которые пересекаются по линии, и (*) является параметрическим уравнением для этой линии.

Пример 7 дает иллюстрацию системы с бесконечным множеством решений, как возникает этот случай и как записывается решение.Каждая линейная система, имеющая бесконечно много решений, должна содержать хотя бы один произвольный параметр (свободная переменная). После того как расширенная матрица была приведена к эшелонированной форме, количество свободных переменных равно общему количеству неизвестных минус количество ненулевых строк:

Это согласуется с теоремой B выше, которая утверждает, что линейная система с меньшим количеством уравнений, чем неизвестных, если она согласована, имеет бесконечно много решений. Условие «меньше уравнений, чем неизвестных» означает, что количество строк в матрице коэффициентов меньше количества неизвестных.Следовательно, приведенное выше уравнение в рамке подразумевает, что должна быть хотя бы одна свободная переменная. Поскольку такая переменная по определению может принимать бесконечно много значений, система будет иметь бесконечно много решений.

Пример 8 : Найти все решения для системы

Во-первых, обратите внимание, что есть четыре неизвестных, но только три уравнения. Следовательно, если система непротиворечива, гарантировано, что у нее будет бесконечно много решений, а это состояние характеризуется по крайней мере одним параметром в общем решении.После того, как соответствующая расширенная матрица построена, исключение Гаусса дает

Тот факт, что в эшелонированной форме расширенной матрицы остаются только две ненулевые строки, означает, что 4-2 = 2 переменных свободны:

Следовательно, выбрав y и z в качестве свободных переменных, пусть y = t 1 и z = t 2 . Во второй строке сокращенной расширенной матрицы следует

, а первая строка дает

Таким образом, решения системы имеют вид

, где т 1 т 2 могут принимать любые реальные значения.

Пример 9 : Пусть b = ( b 1 , b 2 , b 3 ) T и пусть A будет матрицей

Для каких значений b 1 , b 2 и b 3 будет ли система A x = b согласованной?

Расширенная матрица для системы A x = b читает

, который гауссовский элиминатин уменьшает следующим образом:

Нижняя строка теперь подразумевает, что b 1 + 3 b 2 + b 3 должно быть равно нулю, чтобы эта система была согласованной.Следовательно, в данной системе есть решения (фактически бесконечно много) только для тех векторов-столбцов b = ( b 1 , b 2 , b 3 ) T , для которых b 1 + 3 b 2 + b 3 = 0.

Пример 10 : Решите следующую систему (сравните с Примером 12):

Такая система, как эта, где постоянный член в правой части каждого уравнения равен 0, называется однородной системой .В матричной форме он читает A x = 0 . Поскольку каждая однородная система согласована — поскольку x = 0 всегда является решением, — однородная система имеет либо ровно одно решение ( тривиальное решение , x = 0 ), либо бесконечно много. Сокращение строки матрицы коэффициентов для этой системы уже было выполнено в примере 12. Нет необходимости явно дополнять матрицу коэффициентов столбцом b = 0 , поскольку никакая элементарная операция со строкой не может повлиять на эти нули.То есть, если A ‘является эшелонированной формой A , то операции элементарной строки преобразуют [ A | 0 ] в [ A ′ | 0 ]. По результатам Примера 12,

Поскольку последняя строка снова подразумевает, что z можно принять как свободную переменную, пусть z = t , где t — любое действительное число. Обратная подстановка z = t во вторую строку (- y + 5 z = 0) дает

и обратная подстановка z = t и y = 5 t в первую строку ( x + y -3 z = 0) определяет x :

Следовательно, каждое решение этой системы имеет вид ( x, y, z ) = (−2 t , 5 t, t ), где t — любое действительное число.Растворителей бесконечно много, поскольку каждое действительное значение т дает уникальное частное решение.

Обратите внимание на разницу между набором решений для системы в Примере 12 и здесь. Хотя обе имели одинаковую матрицу коэффициентов A , система в Примере 12 была неоднородной ( A x = b , где b 0 ), а здесь — соответствующая однородная система, A x = 0 .Помещая свои решения рядом,

общее решение для Ax = 0 : ( x, y, z ) = (−2 t , 5 t , t )

общее решение для Ax = b : ( x, y, z ) = (−2 t , 5 t , t ) + (−2, 6, 0)

иллюстрирует важный факт:

Теорема C . Общие решения для согласованной неоднородной лиенарной системы, A x = b , равны общему решению соответствующей однородной системы, A x = 0 , плюс частное решение неоднородная система.То есть, если x = x h представляет собой общее решение A x = 0 , то x = x h + x представляет общее решение A x + b , где x — любое конкретное решение (согласованной) неоднородной системы A x = b .

[Техническое примечание: теорема C, которая касается линейной системы , имеет аналог в теории линейных дифференциальных уравнений .Пусть L — линейный дифференциальный оператор; то общее решение разрешимого неоднородного линейного дифференциального уравнения, L (y) = d (где d 0), равно общему решению соответствующего однородного уравнения, L (y) = 0 плюс частное решение неоднородного уравнения. То есть, если y = y h повторно отображает общее решение L (y) = 0, то y = y h + y представляет собой общее решение L (y ) = d , где y — любое частное решение (решаемого) неоднородного линейного уравнения L (y) = d .]

Пример 11 : Определить все решения системы

Запишите расширенную матрицу и выполните следующую последовательность операций:

Поскольку в этой конечной (эшелонированной) матрице остаются только 2 ненулевые строки, есть только 2 ограничения и, следовательно, 4–2 = 2 из неизвестных — например, y и z — являются свободными переменными. Пусть y = t 1 и z = t 2 .Обратная подстановка y = t 1 и z = t 2 во вторую строку ( x — 3 y + 4 z = 1) дает

Наконец, обратная замена x = 1 + 3 t 1 — 4 2 , y = t 1 и z = t 2 в первую строка (2 w -2 x + y = −1) определяет w :

Следовательно, каждое решение этой системы имеет вид

, где t 1 и t 2 — любые вещественные числа.Другой способ написать решение:

, где т 1 , т 2 R .

Пример 12 : Определите общее решение

, которая является однородной системой, соответствующей неоднородной в примере 11 выше.

Поскольку решение неоднородной системы в примере 11 равно

Теорема C означает, что решение соответствующей однородной системы (где t 1 , t 2 R ) получается из (*), просто отбрасывая конкретное решение, x = (1 / 2,1,0,0) неоднородной системы.

Пример 13 : Докажите теорему A: независимо от ее размера или количества неизвестных, содержащихся в ее уравнениях, линейная система не будет иметь решений, ровно одно решение или бесконечно много решений.

Доказательство . Пусть данная линейная система записана в матричной форме A x = b . Теорема действительно сводится к следующему: если A x = b имеет более одного решения, то на самом деле их бесконечно много.Чтобы установить это, пусть x 1 и x 2 будут двумя разными решениями A x = b . Теперь будет показано, что для любого реального значения t вектор x 1 + t ( x 1 x 2 ) также является решением A x = b ; Поскольку t может принимать бесконечно много различных значений, из этого следует желаемый вывод.Начиная с A x 1 = b и A x 2 ,

Следовательно, x 1 + t ( x 1 x 2 ) действительно является решением A x = b , и теорема доказана.

Решение системы с исключением Гаусса

Результаты обучения

  • Используйте метод исключения Гаусса для решения системы уравнений, представленной в виде расширенной матрицы.
  • Интерпретируйте решение системы уравнений, представленное в виде расширенной матрицы.

Мы видели, как написать систему уравнений с расширенной матрицей , а затем как использовать строковые операции и обратную подстановку для получения эшелонированной формы . Теперь мы будем использовать метод исключения Гаусса как инструмент для решения системы, записанной в виде расширенной матрицы. В нашем первом примере мы покажем вам процесс использования исключения Гаусса в системе двух уравнений с двумя переменными.

Пример: решение системы 2 X 2 методом исключения Гаусса

Решите данную систему методом исключения Гаусса.

[латекс] \ begin {array} {l} 2x + 3y = 6 \ hfill \\ \ text {} x-y = \ frac {1} {2} \ hfill \ end {array} [/ latex]

Показать решение

Сначала запишем это как расширенную матрицу.

[латекс] \ left [\ begin {array} {rr} \ hfill 2 & \ hfill 3 \\ \ hfill 1 & \ hfill -1 \ end {array} \ text {} | \ text {} \ begin {array} { r} \ hfill 6 \\ \ hfill \ frac {1} {2} \ end {array} \ right] [/ latex]

Нам нужна 1 в строке 1, столбце 1.Этого можно добиться, поменяв местами строку 1 и строку 2.

[латекс] {R} _ {1} \ leftrightarrow {R} _ {2} \ to \ left [\ begin {array} {rrr} \ hfill 1 & \ hfill -1 & \ hfill \\ \ hfill 2 & \ hfill 3 & \ hfill \ end {array} | \ begin {array} {rr} \ hfill & \ hfill \ frac {1} {2} \\ \ hfill & \ hfill 6 \ end {array} \ right] [/ latex]

Теперь у нас есть 1 в качестве первой записи в строке 1, столбце 1. Теперь давайте получим 0 в строке 2, столбце 1. Это можно сделать, умножив строку 1 на [latex] -2 [/ latex] и затем прибавив результат в строку 2.

[латекс] -2 {R} _ {1} + {R} _ {2} = {R} _ {2} \ to \ left [\ begin {array} {rrr} \ hfill 1 & \ hfill -1 & \ hfill \\ \ hfill 0 & \ hfill 5 & \ hfill \ end {array} | \ begin {array} {rr} \ hfill & \ hfill \ frac {1} {2} \\ \ hfill & \ hfill 5 \ end {массив } \ right] [/ latex]

У нас есть только один шаг, чтобы умножить строку 2 на [latex] \ frac {1} {5} [/ latex].

[латекс] \ frac {1} {5} {R} _ {2} = {R} _ {2} \ to \ left [\ begin {array} {rrr} \ hfill 1 & \ hfill -1 & \ hfill \ \ \ hfill 0 & \ hfill 1 & \ hfill \ end {array} | \ begin {array} {cc} & \ frac {1} {2} \\ & 1 \ end {array} \ right] [/ latex]

Использовать обратную замену.Вторая строка матрицы представляет [латекс] y = 1 [/ латекс]. Подставьте обратно [latex] y = 1 [/ latex] в первое уравнение.

[латекс] \ begin {array} {l} x- \ left (1 \ right) = \ frac {1} {2} \ hfill \\ \ text {} x = \ frac {3} {2} \ hfill \ end {array} [/ latex]

Решение — точка [латекс] \ left (\ frac {3} {2}, 1 \ right) [/ latex].

Попробуй

Решите данную систему методом исключения Гаусса.

[латекс] \ begin {массив} {l} 4x + 3y = 11 \ hfill \\ \ text {} \ text {} \ text {} x — 3y = -1 \ hfill \ end {array} [/ latex]

Показать решение

[латекс] \ влево (2,1 \ вправо) [/ латекс]

В нашем следующем примере мы решим систему двух уравнений с двумя зависимыми переменными.Напомним, что зависимая система имеет бесконечное количество решений, и результатом операций со строками в ее расширенной матрице будет уравнение, такое как [latex] 0 = 0 [/ latex]. Мы также рассмотрим написание общего решения для зависимой системы.

Пример: решение зависимой системы

Решите систему уравнений.

[латекс] \ begin {array} {l} 3x + 4y = 12 \\ 6x + 8y = 24 \ end {array} [/ latex]

Показать решение Выполните строковых операций на расширенной матрице, чтобы попытаться получить -строчную форму .

[латекс] A = \ left [\ begin {array} {llll} 3 \ hfill & \ hfill & 4 \ hfill & \ hfill \\ 6 \ hfill & \ hfill & 8 \ hfill & \ hfill \ end {array} | \ begin {array} {ll} \ hfill & 12 \ hfill \\ \ hfill & 24 \ hfill \ end {array} \ right] [/ latex]

[латекс] \ begin {array} {l} \ hfill \\ \ begin {array} {l} — \ frac {1} {2} {R} _ {2} + {R} _ {1} = { R} _ {1} \ to \ left [\ begin {array} {llll} 0 \ hfill & \ hfill & 0 \ hfill & \ hfill \\ 6 \ hfill & \ hfill & 8 \ hfill & \ hfill \ end { array} | \ begin {array} {ll} \ hfill & 0 \ hfill \\ \ hfill & 24 \ hfill \ end {array} \ right] \ hfill \\ {R} _ {1} \ leftrightarrow {R} _ {2} \ to \ left [\ begin {array} {llll} 6 \ hfill & \ hfill & 8 \ hfill & \ hfill \\ 0 \ hfill & \ hfill & 0 \ hfill & \ hfill \ end {array} | \ begin {array} {ll} \ hfill & 24 \ hfill \\ \ hfill & 0 \ hfill \ end {array} \ right] \ hfill \ end {array} \ hfill \ end {array} [/ latex]

Матрица заканчивается всеми нулями в последней строке: [latex] 0y = 0 [/ latex].Таким образом, существует бесконечное количество решений и система классифицируется как зависимая. Чтобы найти общее решение, вернитесь к одному из исходных уравнений и решите для [latex] y [/ latex].

[латекс] \ begin {array} {l} 3x + 4y = 12 \ hfill \\ \ text {} 4y = 12 — 3x \ hfill \\ \ text {} y = 3- \ frac {3} {4} x \ hfill \ end {array} [/ latex]

Итак, решение этой системы — [латекс] \ left (x, 3- \ frac {3} {4} x \ right) [/ latex].

Теперь мы перейдем на ступенчатую форму, чтобы решить систему линейных уравнений 3 на 3.Общая идея состоит в том, чтобы исключить все переменные, кроме одной, с помощью операций со строками, а затем выполнить обратную замену для поиска других переменных.

Пример: решение системы линейных уравнений с использованием матриц

Решите систему линейных уравнений с помощью матриц.

[латекс] \ begin {массив} {c} \ begin {array} {l} \ hfill \\ \ hfill \\ x-y + z = 8 \ hfill \ end {array} \\ 2x + 3y-z = -2 \\ 3x — 2y — 9z = 9 \ end {array} [/ latex]

Показать решение

Сначала мы пишем расширенную матрицу.

[латекс] \ left [\ begin {array} {rrr} \ hfill 1 & \ hfill -1 & \ hfill 1 \\ \ hfill 2 & \ hfill 3 & \ hfill -1 \\ \ hfill 3 & \ hfill -2 & \ hfill -9 \ end {array} \ text {} | \ text {} \ begin {array} {r} \ hfill 8 \\ \ hfill -2 \\ \ hfill 9 \ end {array} \ right] [/ latex]

Затем мы выполняем операции со строками для получения формы «строка-эшелон».

[латекс] \ begin {array} {rrrrr} \ hfill -2 {R} _ {1} + {R} _ {2} = {R} _ {2} \ to \ left [\ begin {array} { rrrrrr} \ hfill 1 & \ hfill & \ hfill -1 & \ hfill & \ hfill 1 & \ hfill \\ \ hfill 0 & \ hfill & \ hfill 5 & \ hfill & \ hfill -3 & \ hfill \\ \ hfill 3 & \ hfill & \ hfill -2 & \ hfill & \ hfill -9 & \ hfill \ end {array} | \ begin {array} {rr} \ hfill & \ hfill 8 \\ \ hfill & \ hfill -18 \\ \ hfill & \ hfill 9 \ end {массив} \ right] & \ hfill & \ hfill & \ hfill & \ hfill -3 {R} _ {1} + {R} _ {3} = {R} _ {3} \ to \ left [\ begin {array} {rrrrrr} \ hfill 1 & \ hfill & \ hfill -1 & \ hfill & \ hfill 1 & \ hfill \\ \ hfill 0 & \ hfill & \ hfill 5 & \ hfill & \ hfill -3 & \ hfill \\ \ hfill 0 & \ hfill & \ hfill 1 & \ hfill & \ hfill -12 & \ hfill \ end {array} | \ begin {array} {rr} \ hfill & \ hfill 8 \\ \ hfill & \ hfill -18 \\ \ hfill & \ hfill -15 \ end {array} \ right] \ end {array} [/ latex]

Самый простой способ получить 1 в строке 2 столбца 1 — это поменять местами [латекс] {R} _ {2} [/ latex] и [latex] {R} _ {3} [/ latex].

[латекс] \ text {Interchange} {R} _ {2} \ text {и} {R} _ {3} \ to \ left [\ begin {array} {rrrrrrr} \ hfill 1 & \ hfill & \ hfill — 1 & \ hfill & \ hfill 1 & \ hfill & \ hfill 8 \\ \ hfill 0 & \ hfill & \ hfill 1 & \ hfill & \ hfill -12 & \ hfill & \ hfill -15 \\ \ hfill 0 & \ hfill & \ hfill 5 & \ hfill & \ hfill -3 & \ hfill & \ hfill -18 \ end {array} \ right] [/ latex]

Затем

[латекс] \ begin {array} {l} \\ \ begin {array} {rrrrr} \ hfill -5 {R} _ {2} + {R} _ {3} = {R} _ {3} \ в \ left [\ begin {array} {rrrrrr} \ hfill 1 & \ hfill & \ hfill -1 & \ hfill & \ hfill 1 & \ hfill \\ \ hfill 0 & \ hfill & \ hfill 1 & \ hfill & \ hfill -12 & \ hfill \\ \ hfill 0 & \ hfill & \ hfill 0 & \ hfill & \ hfill 57 & \ hfill \ end {array} | \ begin {array} {rr} \ hfill & \ hfill 8 \\ \ hfill & \ hfill -15 \\ \ hfill & \ hfill 57 \ end {array} \ right] & \ hfill & \ hfill & \ hfill & \ hfill — \ frac {1} {57} {R} _ {3} = {R} _ {3} \ to \ left [\ begin {array} {rrrrrr} \ hfill 1 & \ hfill & \ hfill -1 & \ hfill & \ hfill 1 & \ hfill \\ \ hfill 0 & \ hfill & \ hfill 1 & \ hfill & \ hfill -12 & \ hfill \\ \ hfill 0 & \ hfill & \ hfill 0 & \ hfill & \ hfill 1 & \ hfill \ end {array} | \ begin {array} {rr} \ hfill & \ hfill 8 \\ \ hfill & \ hfill -15 \ \ \ hfill & \ hfill 1 \ end {array} \ right] \ end {array} \ end {array} [/ latex]

Последняя матрица представляет собой эквивалентную систему.

[латекс] \ begin {массив} {l} \ text {} x-y + z = 8 \ hfill \\ \ text {} y — 12z = -15 \ hfill \\ \ text {} z = 1 \ hfill \ end {array} [/ latex]

Используя обратную подстановку, мы получаем решение как [latex] \ left (4, -3,1 \ right) [/ latex].

Напомним, что есть три возможных исхода решений для линейных систем. В предыдущем примере решение [латекс] \ left (4, -3,1 \ right) [/ latex] представляет точку в трехмерном пространстве. Эта точка представляет собой пересечение трех плоскостей.В следующем примере мы решаем систему, используя операции со строками, и обнаруживаем, что она представляет зависимую систему. Зависимая система в 3-х измерениях может быть представлена ​​двумя идентичными плоскостями, как в 2-х измерениях, где зависимая система представляет две идентичные линии.

Пример: решение зависимой системы 3 x 3

Решите следующую систему линейных уравнений, используя метод исключения Гаусса.

[латекс] \ begin {array} {r} \ hfill -x — 2y + z = -1 \\ \ hfill 2x + 3y = 2 \\ \ hfill y — 2z = 0 \ end {array} [/ latex]

Показать решение

Запишите расширенную матрицу.

[латекс] \ left [\ begin {array} {rrr} \ hfill -1 & \ hfill -2 & \ hfill 1 \\ \ hfill 2 & \ hfill 3 & \ hfill 0 \\ \ hfill 0 & \ hfill 1 & \ hfill -2 \ end {array} \ text {} | \ text {} \ begin {array} {r} \ hfill -1 \\ \ hfill 2 \\ \ hfill 0 \ end {array} \ right] [/ latex]

Сначала умножьте строку 1 на [latex] -1 [/ latex], чтобы получить 1 в строке 1, столбце 1. Затем выполните операций со строками , чтобы получить форму «строка-эшелон».

[латекс] — {R} _ {1} \ to \ left [\ begin {array} {rrrrrrr} \ hfill 1 & \ hfill & \ hfill 2 & \ hfill & \ hfill -1 & \ hfill & \ hfill 1 \\ \ hfill 2 & \ hfill & \ hfill 3 & \ hfill & \ hfill 0 & \ hfill & \ hfill 2 \\ \ hfill 0 & \ hfill & \ hfill 1 & \ hfill & \ hfill -2 & \ hfill & \ hfill 0 \ end {array} \ справа] [/ латекс]

[латекс] {R} _ {2} \ leftrightarrow {R} _ {3} \ to \ left [\ begin {array} {rrrrr} \ hfill 1 & \ hfill & \ hfill 2 & \ hfill & \ hfill -1 \ \ \ hfill 0 & \ hfill & \ hfill 1 & \ hfill & \ hfill -2 \\ \ hfill 2 & \ hfill & \ hfill 3 & \ hfill & \ hfill 0 \ end {array} \ text {} | \ begin {array} { rr} \ hfill & \ hfill 1 \\ \ hfill & \ hfill 0 \\ \ hfill & \ hfill 2 \ end {array} \ right] [/ latex]

[латекс] -2 {R} _ {1} + {R} _ {3} = {R} _ {3} \ to \ left [\ begin {array} {rrrrrr} \ hfill 1 & \ hfill & \ hfill 2 & \ hfill & \ hfill -1 & \ hfill \\ \ hfill 0 & \ hfill & \ hfill 1 & \ hfill & \ hfill -2 & \ hfill \\ \ hfill 0 & \ hfill & \ hfill -1 & \ hfill & \ hfill 2 & \ hfill \ end {array} | \ begin {array} {rr} \ hfill & \ hfill 1 \\ \ hfill & \ hfill 0 \\ \ hfill & \ hfill 0 \ end {array} \ right] [/ latex]

[латекс] {R} _ {2} + {R} _ {3} = {R} _ {3} \ to \ left [\ begin {array} {rrrrrr} \ hfill 1 & \ hfill & \ hfill 2 & \ hfill & \ hfill -1 & \ hfill \\ \ hfill 0 & \ hfill & \ hfill 1 & \ hfill & \ hfill -2 & \ hfill \\ \ hfill 0 & \ hfill & \ hfill 0 & \ hfill & \ hfill 0 & \ hfill \ end { array} | \ begin {array} {rr} \ hfill & \ hfill 2 \\ \ hfill & \ hfill 1 \\ \ hfill & \ hfill 0 \ end {array} \ right] [/ latex]

Последняя матрица представляет следующую систему.

[латекс] \ begin {array} {l} \ text {} x + 2y-z = 1 \ hfill \\ \ text {} y — 2z = 0 \ hfill \\ \ text {} 0 = 0 \ hfill \ конец {array} [/ latex]

По тождеству [latex] 0 = 0 [/ latex] мы видим, что это зависимая система с бесконечным числом решений. Затем мы находим общее решение. Решив второе уравнение для [latex] y [/ latex] и подставив его в первое уравнение, мы можем решить для [latex] z [/ latex] через [latex] x [/ latex].

[латекс] \ begin {array} {l} \ text {} x + 2y-z = 1 \ hfill \\ \ text {} y = 2z \ hfill \\ \ hfill \\ x + 2 \ left (2z \ справа) -z = 1 \ hfill \\ \ text {} x + 3z = 1 \ hfill \\ \ text {} z = \ frac {1-x} {3} \ hfill \ end {array} [/ latex]

Теперь мы подставляем выражение для [latex] z [/ latex] во второе уравнение, чтобы решить для [latex] y [/ latex] через [latex] x [/ latex].

[латекс] \ begin {массив} {l} \ text {} y — 2z = 0 \ hfill \\ \ text {} z = \ frac {1-x} {3} \ hfill \\ \ hfill \\ y — 2 \ left (\ frac {1-x} {3} \ right) = 0 \ hfill \\ \ text {} y = \ frac {2 — 2x} {3} \ hfill \ end {array} [/ latex ]

Общее решение — [latex] \ left (x, \ frac {2 — 2x} {3}, \ frac {1-x} {3} \ right) [/ latex].

Общее решение для зависимой системы 3 X 3

Напомним, что когда вы решаете зависимую систему линейных уравнений с двумя переменными с использованием исключения или подстановки, вы можете записать решение [latex] (x, y) [/ latex] через x, потому что существует бесконечно много (x, y) пары, которые будут удовлетворять зависимой системе уравнений, и все они попадают на линию [латекс] (x, mx + b) [/ latex].Теперь, когда вы работаете в трех измерениях, решение будет представлять собой плоскость, поэтому вы должны записать его в общей форме [латекс] (x, m_ {1} x + b_ {1}, m_ {2} x + b_ { 2}) [/ латекс].

Попробуй

Решите систему методом исключения Гаусса.

[латекс] \ begin {array} {c} x + 4y-z = 4 \\ 2x + 5y + 8z = 15 \ x + 3y — 3z = 1 \ end {array} [/ latex]

Показать решение

[латекс] \ левый (1,1,1 \ правый) [/ латекс]

Вопросы и ответы

Можно ли решить любую систему линейных уравнений методом исключения Гаусса?

Да, система линейных уравнений любого размера может быть решена методом исключения Гаусса.

Как: решить систему уравнений с помощью матриц с помощью калькулятора

  1. Сохраните расширенную матрицу как матричную переменную [latex] \ left [A \ right], \ left [B \ right], \ left [C \ right] \ text {,} \ dots [/ latex].
  2. Используйте в калькуляторе функцию ref (, вызывая при необходимости каждую матричную переменную.

Пример: решение систем уравнений с помощью калькулятора

Решите систему уравнений.

[латекс] \ begin {array} {r} \ hfill 5x + 3y + 9z = -1 \\ \ hfill -2x + 3y-z = -2 \\ \ hfill -x — 4y + 5z = 1 \ end { array} [/ latex]

Показать решение

Напишите расширенную матрицу для системы уравнений.

[латекс] \ left [\ begin {array} {rrr} \ hfill 5 & \ hfill 3 & \ hfill 9 \\ \ hfill -2 & \ hfill 3 & \ hfill -1 \\ \ hfill -1 & \ hfill -4 & \ hfill 5 \ end {array} \ text {} | \ text {} \ begin {array} {r} \ hfill -1 \\ \ hfill -2 \\ \ hfill 1 \ end {array} \ right] [/ latex]

На странице матриц калькулятора введите расширенную матрицу выше как матричную переменную [latex] \ left [A \ right] [/ latex].

[латекс] \ left [A \ right] = \ left [\ begin {array} {rrrrrrr} \ hfill 5 & \ hfill & \ hfill 3 & \ hfill & \ hfill 9 & \ hfill & \ hfill -1 \\ \ hfill — 2 & \ hfill & \ hfill 3 & \ hfill & \ hfill -1 & \ hfill & \ hfill -2 \\ \ hfill -1 & \ hfill & \ hfill -4 & \ hfill & \ hfill 5 & \ hfill & \ hfill 1 \ end {массив } \ right] [/ latex]

Используйте функцию ref ( в калькуляторе, вызывая матричную переменную [latex] \ left [A \ right] [/ latex].

[латекс] \ text {ref} \ left (\ left [A \ right] \ right) [/ латекс]

Оценить.

[латекс] \ begin {array} {l} \ hfill \\ \ left [\ begin {array} {rrrr} \ hfill 1 & \ hfill \ frac {3} {5} & \ hfill \ frac {9} {5 } & \ hfill \ frac {1} {5} \\ \ hfill 0 & \ hfill 1 & \ hfill \ frac {13} {21} & \ hfill — \ frac {4} {7} \\ \ hfill 0 & \ hfill 0 & \ hfill 1 & \ hfill — \ frac {24} {187} \ end {array} \ right] \ to \ begin {array} {l} x + \ frac {3} {5} y + \ frac {9} {5} z = — \ frac {1} {5} \ hfill \\ \ text {} y + \ frac {13} {21} z = — \ frac {4} {7} \ hfill \\ \ text {} z = — \ frac {24} {187} \ hfill \ end {array} \ hfill \ end {array} [/ latex]

При использовании обратной подстановки решение: [latex] \ left (\ frac {61} {187}, — \ frac {92} {187}, — \ frac {24} {187} \ right) [/ latex] .

Приложения систем уравнений

Теперь обратимся к приложениям, для которых используются системы уравнений. В следующем примере мы определяем, сколько денег было инвестировано по двум разным ставкам, учитывая сумму процентов, полученных на обоих счетах.

Пример: применение матриц 2 × 2 к финансам

Кэролайн инвестирует в общей сложности 12 000 долларов в две муниципальные облигации, одна из которых выплачивает 10,5% годовых, а другая — 12%. Годовой процент, полученный по двум инвестициям в прошлом году, составил 1335 долларов.Сколько было вложено по каждой ставке?

Показать решение

У нас есть система двух уравнений с двумя переменными. Пусть [latex] x = [/ latex] сумма, инвестированная под 10,5% годовых, и [latex] y = [/ latex] сумма, инвестированная под 12% годовых.

[латекс] \ begin {array} {l} \ text {} x + y = 12 000 \ hfill \\ 0.105x + 0.12y = 1335 \ hfill \ end {array} [/ latex]

В качестве матрицы имеем

[латекс] \ left [\ begin {array} {rr} \ hfill 1 & \ hfill 1 \\ \ hfill 0.105 & \ hfill 0.12 \ end {array} \ text {} | \ text {} \ begin {array} { r} \ hfill 12,000 \\ \ hfill 1,335 \ end {array} \ right] [/ latex]

Умножить строку 1 на [латекс] -0.105 [/ latex] и добавьте результат в строку 2.

[латекс] \ left [\ begin {array} {rr} \ hfill 1 & \ hfill 1 \\ \ hfill 0 & \ hfill 0.015 \ end {array} \ text {} | \ text {} \ begin {array} {r } \ hfill 12,000 \\ \ hfill 75 \ end {array} \ right] [/ latex]

Затем,

[латекс] \ begin {array} {l} 0,015y = 75 \ hfill \\ \ text {} y = 5,000 \ hfill \ end {array} [/ latex]

Итак [латекс] 12 000 — 5 000 = 7 000 [/ латекс].

Таким образом, 5000 долларов были инвестированы под 12% годовых и 7000 долларов под 10,5%.

Пример: применение матриц 3 × 3 к финансам

Ava инвестирует в общей сложности 10 000 долларов в три счета, один из которых платит 5% годовых, другой — 8%, а третий — 9%.Годовой процент, полученный по трем инвестициям в прошлом году, составил 770 долларов. Сумма, вложенная под 9%, была вдвое больше, чем сумма, вложенная под 5%. Сколько было вложено по каждой ставке?

Показать решение

У нас есть система трех уравнений с тремя переменными. Пусть [latex] x [/ latex] будет сумма, инвестированная под 5% годовых, пусть [latex] y [/ latex] будет суммой, инвестированной под 8%, и пусть [latex] z [/ latex] будет инвестированной суммой. под 9% годовых. Таким образом,

[латекс] \ begin {array} {l} \ text {} x + y + z = 10 000 \ hfill \\ 0.05x + 0,08y + 0,09z = 770 \ hfill \\ \ text {} 2x-z = 0 \ hfill \ end {array} [/ latex]

В качестве матрицы имеем

[латекс] \ left [\ begin {array} {rrr} \ hfill 1 & \ hfill 1 & \ hfill 1 \\ \ hfill 0,05 & \ hfill 0,08 & \ hfill 0,09 \\ \ hfill 2 & \ hfill 0 & \ hfill -1 \ end {array} \ text {} | \ text {} \ begin {array} {r} \ hfill 10,000 \\ \ hfill 770 \\ \ hfill 0 \ end {array} \ right] [/ latex]

Теперь мы выполняем исключение Гаусса, чтобы получить форму строки-эшелон.

[латекс] \ begin {массив} {l} \ begin {array} {l} \ hfill \\ -0.05 {R} _ {1} + {R} _ {2} = {R} _ {2} \ to \ left [\ begin {array} {rrrrrr} \ hfill 1 & \ hfill & \ hfill 1 & \ hfill & \ hfill 1 & \ hfill \\ \ hfill 0 & \ hfill & \ hfill 0.03 & \ hfill & \ hfill 0.04 & \ hfill \\ \ hfill 2 & \ hfill & \ hfill 0 & \ hfill & \ hfill -1 & \ hfill \ end {array} | \ begin {array} {rr} \ hfill & \ hfill 10,000 \\ \ hfill & \ hfill 270 \\ \ hfill & \ hfill 0 \ end {array} \ right] \ hfill \ end {array} \ hfill \\ -2 {R} _ {1} + {R} _ {3} = {R} _ {3} \ to \ left [\ begin {array} {rrrrrr} \ hfill 1 & \ hfill & \ hfill 1 & \ hfill & \ hfill 1 & \ hfill \\ \ hfill 0 & \ hfill & \ hfill 0.03 & \ hfill & \ hfill 0.04 & \ hfill \\ \ hfill 0 & \ hfill & \ hfill -2 & \ hfill & \ hfill -3 & \ hfill \ end {array} | \ begin {array} {rr} \ hfill & \ hfill 10,000 \\ \ hfill & \ hfill 270 \\ \ hfill & \ hfill -20,000 \ end {array} \ right] \ hfill \\ \ frac {1} {0.03} {R} _ {2} = {R} _ {2} \ to \ left [\ begin {array} {rrrrrr} \ hfill 0 & \ hfill & \ hfill 1 & \ hfill & \ hfill 1 & \ hfill \\ \ hfill 0 & \ hfill & \ hfill 1 & \ hfill & \ hfill \ frac {4} {3} & \ hfill \\ \ hfill 0 & \ hfill & \ hfill -2 & \ hfill & \ hfill -3 & \ hfill \ end {array} | \ begin {array} {rr} \ hfill & \ hfill 10,000 \\ \ hfill & \ hfill 9,000 \\ \ hfill & \ hfill -20,000 \ end {array} \ right] \ hfill \\ 2 {R} _ {2} + {R} _ {3} = {R} _ {3} \ to \ left [\ begin {array} {rrrrrr} \ hfill 1 & \ hfill & \ hfill 1 & \ hfill & \ hfill 1 & \ hfill \\ \ hfill 0 & \ hfill & \ hfill 1 & \ hfill & \ hfill \ frac {4} {3} & \ hfill \\ \ hfill 0 & \ hfill & \ hfill 0 & \ hfill & \ hfill — \ frac {1} {3} & \ hfill \ end {array} | \ begin {array} {rr} \ hfill & \ hfill 10,000 \\ \ hfill & \ hfill 9,000 \\ \ hfill & \ hfill -2,000 \ end {array} \ right] \ hfill \ end {array} [/ latex]

Третья строка сообщает нам [латекс] — \ frac {1} {3} z = -2,000 [/ latex]; таким образом [латекс] z = 6,000 [/ латекс].

Вторая строка сообщает нам [латекс] y + \ frac {4} {3} z = 9000 [/ latex].

Подставляя [латекс] z = 6,000 [/ latex], получаем

[латекс] \ begin {array} {r} \ hfill y + \ frac {4} {3} \ left (6000 \ right) = 9000 \\ \ hfill y + 8000 = 9000 \\ \ hfill y = 1000 \ end {array} [/ latex]

Первая строка сообщает нам [латекс] x + y + z = 10,000 [/ latex]. Подставляя [latex] y = 1,000 [/ latex] и [latex] z = 6,000 [/ latex], мы получаем
[latex] \ begin {array} {l} x + 1,000 + 6,000 = 10,000 \ hfill \\ \ text {} x = 3,000 \ text {} \ hfill \ end {array} [/ latex]

Ответ: 3000 долларов вложены под 5%, 1000 долларов вложены под 8% и 6000 долларов вложены под 9%.

Попробуй

Небольшая обувная компания взяла ссуду в размере 1 500 000 долларов на расширение своего ассортимента. Часть денег была взята под 7%, часть — под 8%, часть — под 10%. Сумма займа под 10% в четыре раза превышала сумму займа под 7%, а годовая процентная ставка по всем трем займам составляла 130 500 долларов. Используйте матрицы, чтобы найти сумму займа по каждой ставке.

Показать решение

150 000 долларов США под 7%, 750 000 долларов США под 8%, 600 000 долларов США под 10%

Внесите свой вклад!

У вас была идея улучшить этот контент? Нам очень понравится ваш вклад.

Улучшить эту страницуПодробнее

ИСКЛЮЧЕНИЕ ПО ГАУССУ: РЕШЕНИЕ СИСТЕМ ЛИНЕЙНЫХ УРАВНЕНИЙ: ПРИМЕРЫ И РЕШАЕМЫЕ ЗАДАЧИ: ВЫСШАЯ ШКОЛА

Содержимое этой страницы:


Введение

Система уравнений (линейная) — это группа (линейных) уравнений с различные неизвестные факторы. Вообще говоря, неизвестные факторы появляются в различных уравнениях.

Уравнение с различными неизвестными факторами связывает их друг с другом.

Решение системы состоит в нахождении значения неизвестных факторов способом, который проверяет все уравнений, составляющих систему.

  • Если существует одно решение (одно значение для каждого неизвестного фактора), мы будем говорить, что система Согласованная независимая система (СНГ) .

  • Если существуют различные решения (система имеет бесконечно много решений), мы говорим, что система является Согласованная зависимая система (CDS)..

  • Если решения нет, а это произойдет, если есть два или несколько уравнений, которые нельзя проверить одновременно, мы говорим, что это несогласованная система (IS) . Например, следующая система уравнений

    $$ \ begin {case} \ begin {array} {lcl} y & = & 0 \\ 2x + y & = & 0 \\ 2x + y & = & 2 \ end {array} \ end {ases} $ $

    несовместимо, потому что мы получаем решение x = 0 из второго уравнения и, из третьего, x = 1 .

В этом разделе мы собираемся решать системы с использованием метода исключения Гаусса , который заключается в простом выполнении элементарных операций в строке или столбце расширенной матрицы, чтобы получить свой эшелон из или его пониженный эшелон форма (Гаусс-Джордан).


Метод разрешения

  1. Применяем метод исключения Гаусса-Иордана : получаем сокращенного эшелона строки формы из расширенной матрицы систему уравнений, выполняя элементарные операции в строках (или столбцах).

  2. Получив матрицу, мы применяем теорему Руше-Капелли для определения тип системы и получить решение (я):

    Пусть A · X = B будет системой из m линейных уравнений с n неизвестными факторы, m и n натуральные числа (не ноль):

    • AX = B соответствует тогда и только тогда, когда,

      $$ ранг (A) = ранг (A | B) $$

    • AX = B — это непротиворечивый независимый тогда и только тогда, когда,

      $$ ранг (A) = n = ранг (A | B) $$

Примечание: Элементарные операции в строках или столбцах позволяют получить системы, эквивалентные исходной, но с формой, упрощающей получение решений (если есть).Также есть более быстрые инструменты для выработки решений в СНГ, такие как правило Крамера.




Система 1

Показать решение

Расширенная матрица системы

того же размера, что и система (2×3). Вертикальная линия, отделяющая матричные коэффициенты от вектора независимых членов.

Выполняем элементарные операции в строках для получения приведенной формы эшелона строк:

Умножаем первую строку на 1/5 а вторую на 1/3

Добавляем вторую строку с первой

Вторую строку умножаем на 5/7

Складываем первую строку со второй, умноженной на -2/5

Эта последняя эквивалентная матрица представлена ​​в сокращенной форме эшелона строк. и это позволяет нам быстро увидеть рейтинг матрица коэффициентов и расширенная.

Рассчитываем ранги:

По теореме Руше-Капелли система непротиворечива Независимая. Полученная матрица представляет собой систему

, который является решением исходной системы.



Система 2

Показать решение

Расширенная матрица системы

Проводим элементарные операции в строках для получения приведенных форма эшелона строки:

Вторую строку умножаем на 1/2

Добавляем первую строку со второй

Умножаем первую строку на 1/3

Эта последняя эквивалентная матрица находится в форме сокращенного эшелона строк и имеет нулевую строку, что означает, что строки в исходной системе линейно зависимы. (любой из них может быть получен путем умножения другого на скаляр, не равный нулю).

Рассчитываем ранги

По теореме Руше-Капелли система непротиворечива. Кроме того, это зависит, потому что ранг (1) ниже, чем количество неизвестных факторов (2).

Полученная матрица представляет собой систему

Решения



Система 3

Показать решение

Расширенная матрица системы

Выполняем элементарные операции в строках, чтобы получить сокращенный рядный эшелон формы

Изменяем порядок строк (так у нас уже будет 1 в первой строке без необходимости умножать)

Складываем вторую строку с первой, умноженной на 5 :

Вторую строку умножаем на -1/15

Эта последняя матрица имеет эшелонированную форму (не сокращена).

Мы можем непосредственно заметить, что система несовместима, потому что у нас следующее равенство (вторая строка):

$$ 0x + 0y = 1 $$

, это невозможное равенство.

Рассчитываем ранги матрицы коэффициентов и дополненной:

По теореме Руше-Капелли система несовместна (решения нет). Полученная матрица представляет собой систему

Система непоследовательна, потому что мы имеем невозможное равенство

$$ 0 = 1 $$



Система 4

Показать решение

Расширенная матрица системы

(размер 3х4).

Выполняем элементарные операции в строках, чтобы получить сокращенный рядный эшелон формы

Умножаем первую строку на 1/5

Складываем вторую и третью строки, умножив первую на -2

Вторую и третью строки умножаем на 5

Складываем вторую строку с третьей, умноженной на -1

Умножаем вторую строку на -1/10 и третью на 1/11

Складываем первую строку со второй, умноженной на -2/5 , а третью со второй, умноженной на -1

Третью строку умножаем на -11/5

Эта последняя эквивалентная матрица имеет сокращенную форму эшелона строк (мы знаем ее, потому что это единичная матрица).Имея единичную матрицу, мы знаем, что это непротиворечивая независимая система, и можем получить единственное решение.

Рассчитываем ранги

По теореме Руше-Капелли система непротиворечива Независимая. Полученная матрица представляет собой систему

, который является решением системы.



Система 5

Показать решение

Расширенная матрица системы

Выполняем элементарные операции в строках, чтобы получить сокращенный рядный эшелон формы

Мы вычитаем первую строку из второй и добавляем третью строку с первой.

Умножаем первую строку на 1/2 , а вторую на 1/3

Мы складываем первую строку со второй, умноженной на 1/2 , и третью строку со второй, умноженной на -2

Третью строку умножаем на 1/3

Складываем первую строку с третьей, умноженной на -3/2

Эта последняя матрица является эшелонированной матрицей с сокращенным числом строк (мы знаем это, потому что у нас есть единичная матрица).

Рассчитываем ранги

По теореме Руше-Капелли система непротиворечива Независимая. Полученная матрица представляет собой систему



Система 6

Показать решение

Расширенная матрица системы

Выполняем элементарные операции в строках, чтобы получить сокращенный рядный эшелон формы

Умножаем вторую строку на -1/3 и третью на 1/4

Складываем вторую и третью строки, умножив первую на -1

Умножаем вторую строку на -3/4 и третью на -1

Третью строку складываем со второй, умноженной на -1

Эта последняя матрица имеет эшелонированную форму (но не сокращена) и мы не продолжаем выполнять операции по строкам, потому что можем видим, что последняя строка делает систему непоследовательной.Эта строка сообщает нам:

$$ 0x + 0y + 0z = -1 $$

И все это невозможное равенство.

Фактически у нас есть ранги

По теореме Руше-Капелли система несовместна.



Система 7

Показать решение

Расширенная матрица системы

Выполняем элементарные операции в строках, чтобы получить сокращенный рядный эшелон формы

Умножаем первую и вторую строки на 1/3 и 1/6 соответственно

Вторую строку вычитаем из третьей

Добавляем первую строку со второй

Третью строку умножаем на 3

Эта последняя матрица находится в сокращенном эшелоне строк формы , поэтому мы можем легко вычислить ранги:

Рассчитываем ранги

По теореме Руше-Капелли система непротиворечива.Но он не является независимым, потому что количество неизвестных факторов (3) отличается от ранга. Полученная матрица представляет собой систему

Решения



Система 8

Показать решение

Расширенная матрица системы

Примечание: прежде, чем мы начнем, мы должны прокомментировать, что процедура будет таким же, как и до сих пор.Но у нас есть корни в матрице, а это значит, что операции в строках будут длинными и утомительно. Эта задача не очень интересна в дидактическом смысле, помимо расчетов.

Проводим элементарные операции в строках, чтобы получить пониженный рядный эшелон формы

Мы складываем вторую строку с первой, умноженной на -√5 и, с третья, умноженная на -2/5

Умножаем вторую строку на 1 / √5 и третью на 5/17

Складываем первую строку с третьей и со второй вычитаем третью

Переписываем матрицу

Умножаем вторую строку на (√5 / 5-5) -1

Складываем первую строку со второй, умноженной на -5

В этой последней матрице она (почти) в строке уменьшена форма эшелона (надо поменять второй и третий ряды так что это правда).Из последней матрицы получаем решения:



Система 9

Показать решение

Расширенная матрица системы

Выполняем элементарные операции в строках, чтобы получить сокращенный рядный эшелон формы

Складываем третью и четвертую строки, умножая первую на 3 и на -2 соответственно

Складываем первую, вторую и третью строки с четвертой, умноженной на 2 , -3 и 5 соответственно

Умножаем четвертую строку на -1 и меняем ее на вторую

Умножаем третью и четвертую строки на 1/34 и -1/22 соответственно

Вычитаем третью строку из четвертой

Умножаем четвертую на -187/42

К первой строке прибавляем третью умноженную на -13 и вторую на 8

К первой строке прибавляем четвертую, умноженную на 5/34 , ко второй прибавляем первую, умноженную на 5/17 и к третьему добавляем первое, умноженное на -3/34

По теореме Руше-Капелли система непротиворечива Независима, и решение равно



Система 10

Показать решение

Расширенная матрица системы

Примечание: Эта система была включена в цель отметить, что теория матрицы применима к комплексным числам.Единственное отличие от предыдущих систем в том, что теперь нам нужно действовать путем умножения и деления комплексные числа.

Выполняем элементарные операции в строках для получения приведенной формы эшелона строк

Умножаем вторую строку на 1/2 и меняем ее на первую

Складываем вторую строку с первой, умноженной на — (1 + i)

Вторую строку умножаем на

Прибавляем первую строку ко второй умноженной на -i / 2

Эта последняя матрица имеет сокращенную форму, поскольку она является единичной матрицей.

По теореме Руше-Капелли система непротиворечива Независима, и решение равно



Matesfacil.com от J. Llopis под лицензией творческий Международная лицензия Commons Attribution-NonCommercial 4.0.


Уловка Гаусса — семинар для сотрудников

Начало работы

Можете ли вы сложить первые 10 чисел в уме? А как насчет первых 100 или первой тысячи? В твоей голове!

Карл Фридрих Гаусс был специальным математиком.История гласит, что в школе, в возрасте 8 лет, он очень быстро сумел сложить первые 100 чисел. Мне нравится думать, что учитель использовал этот трюк много раз, чтобы занять класс надолго, пока он вздремнул. Он знал, что его ждет долгий период затишья, пока класс не работает. Даже если один из них получил ответ, учитель мог попросить его проверить его, чтобы отнять больше времени. Но он не стал торговаться с этим не по годам развитым восьмилетним мальчиком.

В мгновение ока Гаусс выдал 5050.Но он мог не только так быстро вычислить сумму первых 100 чисел, но и обосновать правильность своего ответа. И вы сделаете это до того, как проведете этот семинар для сотрудников.

Возможно, вы захотите прочитать о Карле Фридрихе на одном из многих веб-сайтов. Стоит кое-что записать о Гауссе. Например, где он жил, когда жил, какие бытовые проблемы у него были и тому подобное. Стоит достать карту современной Германии и показать, где находится Брауншвейг.Насколько я помню, это недалеко от Ганновера и старой границы между Восточной и Западной Германией.

Так в чем же секрет и как с его помощью впечатлить друзей и коллег?

Пример 1

Сначала я с трудом сложу целые числа от 1 до 10, чтобы вы могли увидеть, как все работает. Предположим, что сумма первых 10 чисел равна S. Тогда

S = 1 + 2 + 3 + 4 + 5 + 6 + 7 + 8 + 9 + 10.

Интересно то, что если сложить числа в обратном направлении, мы получим тот же ответ.Что ж, очевидно! Но давайте все равно сделаем это.

S = 10 + 9 + 8 + 7 + 6 + 5 + 4 + 3 + 2 + 1.

И что? Что ж, я сделаю так, чтобы «ну и что?» Было легче увидеть, поместив эти два способа написания S друг под другом.

S = 1 + 2 + 3 + 4 + 5 + 6 + 7 + 8 + 9 + 10.

S = 10 + 9 + 8 + 7 + 6 + 5 + 4 + 3 + 2 + 1.

Теперь просто добавьте S к S. Я знаю, что мы, кажется, уходим еще дальше от значения S, которое мы так стремимся получить, но терпите меня. Что ты видишь? Какие закономерности начинают проявляться?

К счастью для Гаусса и нас,

1 + 10 = 2 + 9 = 3 + 8 = 4 + 7 = 5 + 6 = 6 + 5 = 7 + 4 = 8 + 3 = 9 + 2 = 10 + 1 = 11.

Сумма всех этих пар чисел дает 11! Это означает, что

2S = 11 + 11 + 11 + 11 + 11 + 11 + 11 + 11 + 11 + 11.

Вот , у будет десять 11, так что

2S = 10 × 11 = 110.

Итак, S = 5 × 11 = 55.

Но этот трюк нельзя повторять снова и снова. Так что мы его доим изо всех сил.

Пример 2

Давайте сложим числа Гаусса, все целые числа от 1 до 100.Снова позвольте S быть этой суммой. Итак, S = 1 + 2 + 3 + 4 + 5 +… + 98 + 99 + 100.

Теперь вы видите, что я был довольно ленив и опустил все числа от 6 до 97. Но мы с вами знаем, что они действительно есть. Многоточие (…) говорит нам об этом.

О очередь!

S = 100 + 99 + 98 +… + 5 + 4 + 3 + 2 + 1.

Теперь давайте объединим эти две вещи и посмотрим, что произойдет.

S = 1 + 2 + 3 + 4 + 5 +… + 98 + 99 + 100.

S = 100 + 99 + 98 +… + 5 + 4 + 3 + 2 + 1.

Здесь магическая сумма равна 101. Каждая пара чисел, одна над другой, добавляет к 101. Итак, 2S = 101 + 101 + 101 +… + 101 + 101 + 101.

Единственная проблема, которая у нас сейчас есть, — это подсчитать, сколько там 101. Но это не должно быть проблемой. В конце концов, мы начали со 100 чисел, поэтому у нас должно быть 100 сумм, которые складываются с 101. Итак, 2S = 100 × 101.

Это означает, что S = 50 × 101 = 5050.

И Гаусс опередил нас всего на столетие или два.

Теперь вы видите быстрый способ сложить первые 1000 целых чисел? Как насчет первых 10 000, первых 100 000 или первого миллиона?

Пример 3

Я приведу еще один последний пример, прежде чем мы сделаем то, что делает каждый хороший математик, а именно попытаемся обобщить то, что мы делали. Другими словами, мы попытаемся найти закономерность. А пока давайте сложим первые 67 целых чисел.

S = 1 + 2 + 3 +… + 65 + 66 + 67.

S = 67 + 66 + 65 +… + 3 + 2 + 1.

На этот раз ключ — 68. В конце концов, 1 + 67 = 68 = 2 + 66 = 3 + 65 =…

Итак, 2S = 68 + 68 + 68 +… + 68 + 68 + 68.

Затем мы снова сталкиваемся с попыткой вычислить, сколько всего этих сумм. Но мы начали с шестидесяти семи чисел, так что у нас должно быть шестьдесят семь 68. Итак, 2S = 67 × 68, или S = ​​67 × 34 = 2278.

Есть какие-нибудь предположения относительно общей картины здесь?

Обобщение

Я полагаю, что у нас должно быть достаточно информации, чтобы найти сумму первых n целых чисел, где n — любое значение, которое нам нравится.Давайте посмотрим, что нам нужно, чтобы увидеть, сможем ли мы сделать предположение, предположение о том, что происходит на самом деле.

Мы начали с n = 10 и получили S = ​​10 × 11 ÷ 2;

, тогда n = 100 дало нам S = 100 × 101 ÷ 2;

, то n = 67 дает нам S = 67 × 68 ÷ 2.

Похоже, нам нужно взять число, которое мы хотим суммировать, умножить на это число плюс 1, а затем разделить на 2.Итак, у нас

Гипотеза 1: Сумма S первых n чисел равна S = (n x (n +1)) / 2.

Можем ли мы это оправдать, доказать?

Хорошо, пусть S будет суммой чисел от 1 до n, независимо от n.

Если ваша алгебра немного заржавела, замените n ниже на «любое число», измените n — 1 на «любое число минус один», измените n + 1 на «любое число плюс один» и так далее.

Проверенным методом получаем

S = 1 + 2 + 3 +… + (n — 2) + (n — 1) + n.

S = n + (n — 1) + (n — 2) +… + 3 + 2 + 1.

Итак, делая то, что теперь естественно, мы получаем

2S = (n + 1) + (n + 1) + (n + 1) +… + (n + 1) + (n + 1) + (n + 1).

Поскольку вначале было n чисел, теперь должно быть n партий (n + 1). Итак

2S = n × (n + 1).

Итак, S = (n x (n + 1)) / 2.

Похоже, мы опровергли эту гипотезу. Прежде чем продолжить, вы можете подумать над следующими вопросами.

(i) Дает ли эта формула правильный ответ, если n = 15?

(ii) Конечно, S должно быть целым числом, поскольку мы складываем первые n целых чисел. Но мы делим на 2 в правой части уравнения. Может ли n × (n + 1) иногда быть нечетным и все портить?

(iii) Что эта формула говорит словами?

Немного дальше

Но вам не обязательно добавлять только первые числа. Предположим, мы хотим сложить все числа от 8 до 93.Как мы могли это сделать?

Мне кажется, что мы могли бы сделать это по крайней мере тремя способами, но я не буду беспокоиться о том, когда вы складываете числа по одному.

Метод 1: Мы могли бы записать числа от 8 до 93 в обычном порядке, а затем записать их в обратном порядке, как мы это делали в других примерах. Я предоставлю вам сделать это, чтобы посмотреть, что у вас получится.

Method 2: С другой стороны, мы могли бы сначала прибавить 1 к 7, а затем от 1 к 93, используя нашу формулу.Тогда мы сможем вычесть меньшее из большего. Как это:

В 1 + 2 +… + 6 + 7, «любое число», n равно 7, поэтому сумма этих чисел составляет (7 x 8) / 2 = 28.

В 1 + 2 +… + 92 + 93, «любое число», n равно 93, поэтому сумма этих чисел составляет (93 x 94) / 2 = 4371.

Итак, нам нужна сумма 4371 — 28 = 4343.

Прежде чем продолжить, вы можете подумать над следующими вопросами.

(iv) Существует ли формула для суммы чисел от любого числа, которое вы выберете (например, 8), до любого другого числа, которое вы выберете (например, 93)? Другими словами, можете ли вы обобщить гипотезу?

(v) Вы видите, как мы медленно попадаем в более сложные ситуации? Это путь, по которому математика всегда пытается расширить наши знания о мире.

(vi) В свете мысли в (v), куда мы должны двигаться дальше? Каким будет следующий способ расширить то, что мы делаем? Мы пробовали переходить от 1 к чему-то, а затем от чего-то к чему-то еще, но шаги от числа к числу всегда были единицами. Можем ли мы добиться прогресса, если ступеньки больше единицы?

(vii) В конце концов, существует ли только одна формула для ряда сложений, которые не просто складывают первое такое количество чисел? Что могла бы быть эта формула на словах?

Семинар

Еще раз вам придется подумать о том, как представить этот материал, который лучше всего подходит для ваших сотрудников, но как насчет следующего?

Установите их, задав им вопрос, который задал ему учитель Гаусса.Пусть они поработают немного. Затем дайте им понять, что 8-летний ребенок может сделать это в своей голове. Это должно привести к поиску некоторых закономерностей в числах от 1 до 100, которые могут облегчить быстрое суммирование. Например, некоторые группы видят, что 1 + 100 = 2 + 99 и так далее. Обычно они не думают о сложении двух сумм S. Но вы можете быстро сложить числа от 1 до 100 и другим способом.

Тогда попробуйте их с другими примерами. Если вы вооружитесь калькулятором, вы можете предложить им сложить числа от 1 ко всему, что они выберут, быстрее, чем вы.

Тогда им следует подумать, что вы знаете, чего они не знают? Попросите их сделать несколько примеров и предположить, что это за образец.

В зависимости от того, насколько хорошо дела обстоят, вы можете перейти к некоторым арифметическим прогрессиям, где общая разница не равна 1 (см. Раздел 8). С заинтересованной группой вы могли бы даже провести доказательство.

Но вы должны сказать кое-что о Гауссе и его значении на математической сцене. Вы также должны найти несколько интересных историй о нем по ссылке, которую я дал выше.Немного истории никогда не заблудится.

Ответы на некоторые вопросы

В этом разделе мы завершаем работу, которую мы проделали в разделах 2–6. Конечно, насколько далеко вы зайдете с этой проблемой, будет зависеть от алгебраической уверенности ваших сотрудников, хотя вы можете полностью обойти алгебру, если думаете, что она пойдет. как свинцовый шар. В любом случае, постарайтесь немного вытолкнуть их из зоны комфорта, но бросьте им спасательный круг, когда они тонут. Мы оставляем это решение на ваше усмотрение, но здесь должно быть достаточно материала для вашего семинара.

Сейчас я попытаюсь найти формулу для суммы строки чисел, в которой шаг вверх от одного числа к другому всегда одинаков. Приведу два примера, а затем решу задачу в целом.

Первый пример: Суммируйте числа 2 + 4 + 6 +… + 64 + 66 + 68.

Это можно сделать несколькими способами. Прямое сложение — это единица, как и деление всех чисел в сумме на 2 и использование известной нам формулы. Однако я возвращаюсь к испытанному методу «сначала вперед, затем назад».Итак, пусть S будет суммой, которую мы ищем. Итак, вперед, затем назад, мы имеем

S = 2 + 4 + 6 +… + 64 + 66 + 68.

S = 68 + 66 + 64 +… + 6 + 4 + 2.

Это означает, что 2S = 70 + 70 + 70 +… + 70 + 70 + 70.

Единственная проблема сейчас в том, сколько там терминов? Что ж, если бы мы разделили все исходные числа на 2, у нас было бы 1 + 2 + 3 +… + 32 + 33 + 34. Поскольку здесь 34 члена, должно быть 34 члена в S. Итак, 2S = 34 × 70 и S = ​​(34 x 70) / 2 = 1190.

Вы можете проверить это одним из других методов, но он немного похож на формулу, которую мы нашли в разделе 5.

Второй пример: Суммируйте числа 9 + 12 + 15 +… + 54 + 57 + 60.

Это можно сделать несколькими способами. Прямое сложение равно единице, как и деление всех чисел в сумме на 3 и использование известной нам формулы. Однако я возвращаюсь к испытанному методу «сначала вперед, затем назад». Итак, пусть S будет суммой, которую мы ищем.Итак, вперед, затем назад, мы имеем

S = 9 + 12 + 15 +… + 54 + 57 + 60.

S = 60 + 57 + 54 +… + 15 + 12 + 9.

Это означает, что 2S = 69 + 69 + 69 +… + 69 + 69 + 69.

Единственная проблема сейчас в том, сколько там терминов? Что ж, если бы мы разделили все исходные числа на 3, у нас было бы 3 + 4 + 5 +… + 18 + 19 + 20. Поскольку здесь 20 — 2 = 18 членов, должно быть 18 членов в S. Итак, 2S = 18 × 69 и S = ​​(18 x 69) / 2 = 621.

Вы можете проверить это одним из других методов.Здесь есть закономерность? Эти слова звучат знакомо?

Общий пример: Во-первых, можем ли мы угадать, что мы надеемся найти? Мы хотели бы найти формулу для суммы набора чисел, которые где-то начинаются и попадают в другое место, но при этом шаг между числами всегда одинаков. Исходя из имеющейся информации, можем ли мы угадать, какой могла бы быть формула, прежде чем мы разберемся в беспорядке алгебры, с которым нам придется столкнуться, чтобы получить ответ? В каждом случае, какие два числа мы умножаем, чтобы получить S?

Мы знаем, что когда мы добавили от 1 к n, числа были n и n + 1.Когда мы добавили от 2 до 68, их было 34 и 70; когда мы добавили от 9 до 60, их было 18 и 69.

Очевидно, что в каждом случае большее число — это общая сумма, которую мы получаем, складывая большие числа с меньшими числами. Эти большие числа представляют собой сумму наименьшего и наибольшего чисел.

А как насчет 34 и 18? И как они соотносятся с n, которое мы получили при сложении первых n чисел. Что у них общего? Разве это не просто количество складываемых чисел? Означает ли это, что формула, которую мы должны получить, содержится в следующей гипотезе?

Гипотеза 2: Если S — сумма любой из этих строк, где есть общее различие, S = ((количество членов) (сумма первого и последнего чисел)) / 2

Проверьте формулу на предмет других наборов чисел, которые где-то начинаются и увеличиваются на постоянную величину.Другими словами, наборы чисел, в которых есть общая разница между последовательными числами.

На данном этапе у нас есть предположение относительно того, каким может быть ответ. Это довольно сильное предположение, потому что оно работает на множестве примеров. Но можем ли мы доказать, что это работает для каждого набора чисел с общим свойством разности? Ну конечно можем. И сначала мы покажем это словесным методом, а затем посмотрим, насколько проще выразить то же самое с помощью алгебры.

Предположим, что набор чисел — это некоторое число, первое число; некоторое число плюс общая разница, второе число; некоторое число плюс общее различие плюс общее различие, третье число; до самого большого числа, последнего числа.

Тогда сумму S можно записать двумя обычными способами:

S = первое число + второе число +… + второе последнее число + последнее число

S = последнее число + второе последнее число +… + второе число + первое число.

Теперь первое число + последнее число = второе число + второе последнее число. Это потому, что мы поднимаемся на общую разницу, идущую от первого числа ко второму числу, и вниз на общую разницу, идущую от последнего числа ко второму последнему числу.Итак, как обычно, все отдельные суммы одинаковы. Итак

2S = (первое число + последнее число) + (первое число + последнее число) +… + (первое число + последнее число) + (первое число + последнее число).

Но в скобках указана одна из сумм для каждого слагаемого исходной суммы. Итак, 2S = (количество терминов) (первое число + последнее число), и поэтому S = (количество терминов) (первое число + последнее число) ÷ 2.

Это то, что мы предположили выше. И теперь мы доказали эту гипотезу, и она верна для любого набора чисел, который увеличивается с одинаковыми шагами.Эти наборы называются арифметических прогрессий .

Между прочим, математики работали во многом так же, как мы, до изобретения алгебры. Даже в работе Ньютона вы найдете уравнения со словами. Здесь это неплохо, но может стать очень громоздким. С появлением алгебры математическая жизнь значительно улучшилась.

Если вам нужна полная алгебраическая версия, вот она. Пусть первый член будет a, общая разница будет d, а количество членов будет n.Тогда

S = a + (a + d) + (a + 2d) +… + [a + (n — 3) d] + [a + (n — 2) d] + [a + (n — 1) d ]

S = [a + (n — 1) d] + [a + (n — 1) d] + [a + (n — 1) d] +… + (a + 2d) + (a + d) +

Итак, 2S = [2a + (n — 1) d] + [2a + (n — 1) d] + [2a + (n — 1) d] +… + [2a + (n — 1) d] + [2a + (n — 1) d] + [2a + (n — 1) d]

или S = ​​n [2a + (n — 1) d] ÷ 2.

Одним из преимуществ этого метода является то, что легче увидеть, что сумма каждой пары соответствующих терминов одинакова.Другой заключается в том, что он записывает ответ в терминах первого члена, количества терминов и общей разницы. Единственным недостатком, по-видимому, является то, что он скрывает тот факт, что формула включает «первое число плюс последнее число», записывая это выражение как [2a + (n — 1) d]. И форму, которую мы написали в Гипотезе 2, легче запомнить.

7.3 — Метод исключения

7.3 — Метод исключения

7.3 — Метод исключения

Щелкните здесь, чтобы получить инструкции по использованию Algebra Coach провести метод исключения.

В этом разделе мы объясняем метод исключения. Этот метод использует тот факт, что решение уравнения не меняется, если мы

  • умножьте обе части уравнения на один и тот же коэффициент.
  • вычтите равные величины из обеих частей уравнения.
Это означает, что мы можем взять одно уравнение и вычесть несколько из другого. уравнение из него без изменения решения уравнений.

Метод исключения использует этот факт для решения системы линейных уравнений. Предположим, мы начнем с системы из n уравнений с n неизвестными. Выберите первое уравнение и вычтите из другого подходящие числа, кратные ему. n — 1 уравнение. В каждом случае выбирается кратное так что вычитание отменяет или исключает ту же переменную , скажем x . В результате уравнения n -1 содержат только n — 1 неизвестное ( x больше не отображается).

Мы повторяем этот процесс исключения до тех пор, пока не получим 1 уравнение из 1 неизвестных, которое затем легко решается.

Последний шаг — обратная замена уже полученного решения на 1 неизвестно в предыдущие уравнения, чтобы найти значения всех остальных неизвестных.



Пример: Решите эту систему уравнений методом исключения:
Решение: Давайте дважды возьмем первое уравнение, а именно:
2 x + 2 y = 8
и вычтите его из второго уравнения, например:
В результате получается одно уравнение из одной неизвестной: y .Другой неизвестный, x , был устранено. Решение этого уравнения дает y = 0,4.

Осталось найти х . Если мы обратно подставим y = 0,4 в любое из исходных уравнений получаем х = 3,6. Таким образом, решение:

{ x = 3,6, y = 0,4}.
(Обратите внимание, что вместо этого мы могли бы найти x без обратной подстановки, если бы мы вычитали 3 раза первое уравнение из второго, так как это исключает y .)

Расширенная матрица

Мы объяснили суть исключения. Для более крупных систем нам понадобится систематическая процедура во избежание путаницы. Две такие процедуры — это исключение Гаусса и исключение Гаусса-Жордана.

Прежде чем мы их опишем, приведем краткое описание. Система уравнений, такая как:

будет представлен прямоугольным массивом чисел, называемым Расширенная матрица :

Определения:

  • Отдельные числа в матрице называются элементами .
  • Столбцы проходят вниз по матрице. например столбец 4 содержит элементы 80, 7 и 22.
  • Ряды переходят поперек. Строка 3 ряд содержит 3, −1, 2 и 22. Обратите внимание, что количество столбцов в расширенной матрице всегда на 1 больше. чем количество строк.
  • Диагональ — это набор элементов, который начинается сверху, слева. угол матрицы и проходит по диагонали вниз и вправо.Диагональ указанной выше матрицы состоит из чисел 4, 1 и 2.
  • Любые числа в позиции D считаются по диагонали , любые в позиции a на выше диагонали , а любые в позиции b на ниже диагонали .
Помните следующее:
  • Строка i расширенной матрицы представляет i -е уравнение.
  • Столбец j (слева от вертикальной линии) содержит коэффициенты j -й переменной или неизвестно.
  • Вертикальная линия представляет знаки равенства.
  • Столбец справа от вертикальной линии представляет правую часть уравнений.



Операции элементарных строк

Мы видели, что решение системы уравнений не изменится, если мы:
  • разделите обе части уравнения на константу, или
  • вычесть кратное одного уравнения из другого уравнения.
Эти же операции можно применить к строкам расширенной матрицы, поскольку каждая строка представляет собой просто уравнение. Затем они называются Операции элементарной строки .

Операции элементарной строки (E.R.O.):
  • E.R.O. # 1: Выберите строку расширенной матрицы и разделите (каждый элемент) строки константой.
  • E.R.O. # 2: Выберите любую строку расширенной матрицы и вычтите кратно любой другой строке из него (элемент за элементом).
  • E.R.O. # 3: Иногда бывает полезно поменять местами две строки. Это допустимая операция, потому что порядок уравнений несущественный.
Вы можете применять эти E.R.O. к расширенной матрице так часто, как захотите. без изменения решения уравнений, представленных матрицей.

Пример: В этом примере показано, как мы применяем E.R.O. # 1 и обозначение мы используем, чтобы обозначить это. Разделим первую строку дополненной матрица слева на 2, чтобы получить новую расширенную матрицу справа:

Примечание: ← ÷ на 2 означает « разделить строку указывается цифрой 2 для создания новой матрицы ”.

Пример: В этом примере показано, как мы применяем E.R.O. # 2 и обозначения мы используем, чтобы обозначить это. В расширенной матрице слева мы будем возьмите вторую строку и вычтите из нее 3 раза первую строку, чтобы получить новая расширенная матрица справа:
Примечание: R 2 — 3 · R 1 означает « возьмите строку, на которую указывает (строка 2), и вычтите 3 раза строку 1 из для создания новой строки 2.

Пример: В этом примере показано, как мы применяем E.R.O. # 3 и обозначения мы используем, чтобы обозначить это. В расширенной матрице слева мы меняем местами строки 1 и 2, чтобы получить новая расширенная матрица справа:
Примечание: R 1 R 2 означает « поменять местами строки 1 и 2. »

Исключение по Гауссу

Процедура исключения по Гауссу представляет собой определенную последовательность действий E.R.O. который преобразует расширенную матрицу в форму Гаусса (также известная как эшелонная форма ) Эта форма характеризуется единицей на диагонали, 0 под диагональю. и любые числа выше диагонали.Вот пример:
Эта расширенная матрица представляет система уравнений:
Решается обратной подстановкой. Подключение z = 3 к второе уравнение дает y = 5. Затем вставляем как z = 3, так и y = 5 в первом уравнении дает x = 7.

Алгоритм исключения Гаусса

Мы преобразуем один столбец за раз в форму эшелона строк (или гаусса).Преобразуемый в настоящее время столбец называется ось поворотная . Мы действуем планомерно, позволяя опорному столбцу быть первым столбцом, затем вторым столбцом и т. д. до последнего столбца перед вертикальной чертой расширенной матрицы. Для каждого сводного столбца мы делаем следующие два шага, прежде чем перейти к следующему сводному столбцу:

  1. Размещаем диагональный элемент в опорной колонне. Этот элемент называется осью .Строка, содержащая стержень, — называется опорной строкой . Мы делим каждый элемент в сводной строке на pivot (т.е. мы используем E.R.O. # 1), чтобы получить новую строку pivot с 1 в поворотное положение.
  2. Мы получаем 0 в каждой позиции ниже позиции поворота на вычитание подходящего кратного числа сводной строки из каждого строки под ним (т.е. используя E.R.O. # 2).
Когда все столбцы слева от вертикальной линии были преобразованы с помощью процедура исключения Гаусса расширенная матрица будет в форме Гаусса и затем мы решаем систему обратной подстановкой.

Пример: Используйте метод исключения Гаусса для решения системы уравнений:

Решение: Выполните следующую последовательность E.R.O. на расширенной матрице:

Установите столбец сводной таблицы в столбец 1. Получите 1 в диагональном положении (красным):


Затем установите 0 под точкой поворота (красным):

Теперь пусть сводный столбец = второй столбец.

Сначала получите 1 по диагонали:


Затем получите 0 в позиции под точкой поворота:

Теперь пусть сводный столбец = третий столбец. Получите 1 по диагонали:

Эта матрица, которая теперь имеет форму Гаусса, представляет собой систему трех уравнения:

Решается обратной подстановкой. Вставляем z = 3 во вторую уравнение получаем y = 5.И вставка z = 3 и y = 5 в первое уравнение получаем x = 7. Таким образом, решение:
{ x = 7, y = 5, z = 3}.


Гаусс-Джордан Ликвидация

Процедура исключения Гаусса-Джордана представляет собой несколько иную последовательность E.R.O., который преобразует расширенную матрицу в форму Гаусса-Жордана (также известная как сокращенная форма эшелона ).Эта форма характеризуется единицей на диагонали, 0 над и под диагональю слева от вертикальной линии, и любые числа справа от вертикальной линии. Вот пример:
Эта расширенная матрица представляет собой систему уравнений:
Эта система уже решена: x = 7, y = 5, z = 3. Обратная подстановка не требуется. Однако примерно в два раза больше E.R.O. требуются для производства Форма Гаусса-Жордана как форма Гаусса.

Алгоритм исключения Гаусса-Джордана

Мы преобразуем по одному столбцу за раз в сокращенную форму эшелона строк (или форму Гаусса-Жордана). Преобразуемый в настоящее время столбец называется ось поворотная . Мы действуем планомерно, позволяя опорному столбцу быть первым столбцом, затем вторым столбцом и т. д. до последнего столбца перед вертикальной чертой расширенной матрицы.Для каждого сводного столбца мы делаем следующие два шага, прежде чем перейти к следующему сводному столбцу:

  1. Размещаем диагональный элемент в опорной колонне. Этот элемент называется осью . Строка, содержащая стержень, — называется опорной строкой . Мы делим каждый элемент в сводной строке на pivot (т.е. мы используем E.R.O. # 1), чтобы получить новую строку pivot с 1 в поворотное положение.
  2. Мы получаем 0 в каждой позиции выше и ниже позиции поворота на вычитание подходящего кратного числа сводной строки из каждого ряды над и под ним (т.е. с помощью E.R.O. №2).
Когда все столбцы слева от вертикальной линии будут преобразованы, расширенная матрица будет в форме Гаусса-Жордана, и мы сможем считать решение из правого столбца.

Обратите внимание, что единственное отличие от процедуры Гаусса состоит в том, что на втором этапе мы получаем 0s выше диагонали , а также ниже диагонали. Важно получить все эти нули, прежде чем переходить к следующему сводному столбцу.

Пример: Используйте метод исключения Гаусса-Жордана для решения системы уравнений:
Решение: Выполните эту последовательность E.R.O. на расширенной матрице. Установите для сводного столбца столбец 1. Получите 1 по диагонали. (красным) с помощью E.R.O. №1:
Затем установите 0 под точкой поворота (красным) с помощью E.R.O. # 2:
Теперь пусть сводный столбец = второй столбец.Сначала получите 1 по диагонали. с помощью E.R.O. №1:
Затем получите 0 в позициях выше и ниже точки поворота (красным) с помощью E.R.O. # 2:
Теперь пусть сводный столбец = третий столбец. Получите 1 в диагональном положении, используя E.R.O. №1:
Затем получите 0 в позициях над точкой поворота (красным) с помощью E.R.O. # 2:
Эта матрица, которая теперь имеет форму Гаусса-Жордана или сокращенную форму эшелона строк, представляет собой решение:
{ x = 49, y = −18, z = 8}.



Резервные и несовместимые системы

Если количество уравнений больше, чем количество неизвестных, то системы гарантированно будут резервными или непоследовательно. Но если количество уравнения равно или меньше количества неизвестных, тогда вы, как правило, не распознавать систему как избыточную или несовместимую до самого конца расчета. Это особенно актуально, если система большая.

Если вы решаете систему уравнений методом подстановки и система избыточна, тогда вы получите окончательное уравнение, в котором 0 = 0. Или, если система непоследовательна, вы получите ту, которая утверждает противоречие вида 0 = 5. Нечто подобное происходит при использовании исключения Гаусса или Гаусса-Джордана. Если система резервная, то по окончании процедуры устранения когда у вас есть расширенная матрица в форме Гаусса или Гаусса-Жордана, последняя строка расширенной матрицы будет:

Эта последняя строка представляет уравнение 0 = 0, бесполезную информацию.

Если система несовместима, то последняя строка расширенной матрицы будет выглядеть примерно так:

Последняя строка представляет уравнение 0 = 5, противоречие. Попробуйте упражнения, которые содержат примеры избыточные и противоречивые системы уравнений.

Пример: Используйте метод исключения Гаусса, чтобы преобразовать эту систему уравнений в форму эшелона строк и интерпретировать результат:
Решение: Выполните эту последовательность E.R.O. на расширенной матрице. Установите столбец поворота на столбец 1. В позиции поворота уже стоит 1, поэтому переходите к получению 0 под поворотом:
Теперь установите сводный столбец во второй столбец. В позиции поворота уже есть 1, поэтому переходите к получению 0 под точкой поворота:
Теперь установите сводный столбец в третий столбец. Первое, что нужно сделать, это получить 1 по диагонали, но нет никакого способа сделать это. Фактически эта матрица уже представлена ​​в виде эшелона строк и представляет:
Эта система уравнений противоречива и не имеет решения.Последнее уравнение утверждает противоречие, а именно 0 = −50.
В общем случае расширенная матрица, которая была помещена в форму эшелона строк и которая содержит одну или несколько нижних строк, состоящих из всех нулей слева от вертикальной линии и ненулевого числа справа, указывает на несовместимую систему уравнения без решения.



Уравнений меньше, чем неизвестных

Если количество уравнений в системе меньше количества неизвестных, то вы достигнете точки Гаусса или Гаусса-Иордана процедура, в которой вы не можете преобразовать стержень столбец, потому что у вас закончились сводные строки.Вот пример:
В третьем столбце нет оси и нет строки сводки, поэтому вам нужно остановиться. Эта расширенная матрица представляет эту систему уравнений:
Во второй форме мы видим, что если для z задано значение, то x и y можно выразить через него. Следующая матрица показывает, что предоставление значения для z , скажем z = 5, составляет другую строку:
Попробуйте упражнения, содержащие примеры систем с меньшим количеством уравнений. чем неизвестные.



Если вы нашли эту страницу в поиске в Интернете, вы не увидите
Оглавление в рамке слева.
Щелкните здесь, чтобы отобразить его.

Как использовать метод исключения Гаусса для решения системы уравнений?

ПРИМЕР:

Используйте метод исключения Гаусса для решения следующей системы уравнений.

# x + 2y + 3z = -7 #
# 2x-3y-5z = 9 #
# -6z-8y + z = -22 #

Решение:

Настроить расширенную матрицу формы.

# ((1,2,3, |, -7), (2,3, -5, |, 9), (- 6, -8,1, |, 22)) #

Цель 1. Получите 1 в верхнем левом углу.

Уже сделано.

Цель 2a: Получите ноль под 1 в первом столбце.

Умножьте строку 1 на # -2 #, чтобы получить

# ((- 2, -4, -6, |, 14)) #

Добавьте результат в строку 2 и поместите результат в строку 2.

Обозначим операции как # -2R_2 + R_1 → R_2 #.

# ((1,2,3, |, -7), (2,3, -5, |, 9), (- 6, -8,1, |, 22)) stackrel (-2R_1 + R_2 → R_2) (→) ((1,2,3, |, -7), (0, -7, -11, |, 23), (- 6, -8,1, |, 22)) #

Цель 2b: Получите еще один ноль в первом столбце.

Для этого нам понадобится операция # 6R_1 + R_3 → R_3 #.

# ((1,2,3, |, -7), (0, -7, -11, |, 23), (- 6, -8,1, |, 22)) stackrel (6R_2 + R_3 → R_3) (→) ((1,2,3, |, -7), (0, -7, -11, |, 23), (0,4,19, |, -64)) #

Цель 2c. Получите оставшийся ноль.

Умножьте строку 2 на # -1 / 7 #.

# ((1,2,3, |, -7), (0, -7, -11, |, 23), (0,4,19, |, -64)) stackrel (- (1/7 ) R_2 → R_2) (→) ((1,2,3, |, -7), (0,1,11 / 7, |, -23 / 7), (0,4,19, |, -64 )) #

Теперь используйте операцию # -4R_2 + R_3 → R_3 #.

# ((1,2,3, |, -7), (0,1,11 / 7, |, -23 / 7), (0,4,19, |, -64)) stackrel (-4R_2 + R_3 → R_3) (→) ((1,2,3, |, -7), (0,1,11 / 7, |, -23 / 7), (0,0,89 / 7, |, -356/7)) #

Умножьте третью строку на # 7/89 #.

# ((1,2,3, |, -7), (0,1,11 / 7, |, -23 / 7), (0,0,89 / 7, |, -356 / 7)) stackrel (7 / 89R_3 → R_3) (→) ((1,2,3, |, -7), (0,1,11 / 7, |, -23 / 7), (0,0,1, | , -4)) #

Цель 3. Используйте обратную замену, чтобы получить значения # x #, # y # и # z #.

Цель 3а. Рассчитайте # z #.

#z = -4 #

Цель 3b. Вычислить # y #.

# y + 11 / 7z = -23 / 7 #
# y-44/7 = -23 / 7 #
# y = 44 / 7-23 / 7 = 21/7 #

# у = 3 #

Цель 3c. Вычислить x.

# x + 2y + 3z = -7 #
# x + 6-12 = -7 #
# x-6 = -7 #

# х = 1 #

Решение: # x = 1, y = 3, z = -4 #

Методы исключения Гаусса и матричные методы

Методы исключения Гаусса и матричные методы





Система линейных уравнений может быть помещены в матричную форму.Каждый уравнение становится строкой, и каждое переменная становится столбцом. An добавлен дополнительный столбец для справа. Система показаны линейные уравнения и результирующая матрица.

Система линейных уравнений …

 3x + 2y - 4z = 3
2х + 3у + 3z = 15
5х - 3у + г = 14 

становится расширенной матрицей …

х y z rhs
3 2-4 3
2 3 3 15
5 -3 1 14

Цель при решении системы уравнений состоит в том, чтобы по возможности преобразовать расширенную матрицу в сокращенную форму строки-эшелона.

Есть три элементарные операции со строками, которые вы можете использовать для размещения матрицы в приведенная строчно-эшелонированная форма.

Каждое из требований сокращенной матрицы строка-эшелон может быть удовлетворено с использованием элементарной строки операции.

  • Если есть строка со всеми нулями, то она находится внизу матрицы.
    Поменяйте местами две строки матрицы, чтобы переместить строку со всеми нулями вниз.
  • Первый ненулевой элемент любой строки — это единица.Этот элемент называется ведущим.
    Умножьте (разделите) строку на ненулевую константу, чтобы превратить первый ненулевой элемент в один.
  • Первая строка любой строки находится справа от первой строки предыдущей строки.
    Умножьте строку на ненулевую константу и добавьте ее в другую строку, заменив эту строку. В Смысл этой элементарной операции со строками состоит в том, чтобы преобразовать числа в нули. Сделав числа под ведущими в ноль, это заставляет первый ненулевой элемент любой строки быть справа от ведущей предыдущей строки.
  • Все элементы выше и ниже ведущего равны нулю.
    Умножьте строку на ненулевую константу и добавьте ее в другую строку, заменив эту строку. В Смысл этой элементарной операции со строками — преобразовать числа в ноль. Разница здесь в что вы очищаете (обнуляете) элементы выше ведущего, а не чуть ниже ведущий.

Что такое поворот?

Цель поворота — сделать элемент выше или ниже ведущего. в ноль.

«Поворотный элемент» или «сводный элемент» — это элемент в левой части матрицы. что вы хотите элементы сверху и снизу равны нулю.

Обычно это единица. Если вы найдете книгу, в которой упоминается поворот, они обычно сказать вам, что вы должны повернуться на один. Если ограничиться тремя элементарными рядами операций, то это верное утверждение.

Однако, если вы хотите объединить вторую и третью элементарные операции со строками, вы придумать другую строковую операцию (не элементарную, но все еще действующую).

  • Вы можете умножить строку на ненулевую константу и добавить ее к ненулевому кратному другому row, заменив эту строку.

И что? Если вам нужно повернуться на одном, то вам иногда придется использовать второй. элементарная операция со строкой и разделите строку на ведущий элемент, чтобы превратить ее в единицу. Деление приводит к дробям. Хотя дроби — ваши друзья, у вас меньше шансов ошибиться если вы их не используете.

В чем прикол? Если вы не остановитесь на одном, вы, вероятно, столкнетесь с большими числами. Большинство люди готовы работать с большими числами, чтобы избежать дробей.

Процесс поворота

Pivoting работает, потому что общее кратное (не обязательно наименьшее общее кратное) двух чисел всегда можно найти, умножив два числа вместе. Возьмем предыдущий пример и очистить первый столбец.

х y z rhs
3 2-4 3
2 3 3 15
5 -3 1 14

Полезные советы

  • Хотя вам не нужно поворачиваться на одном, это очень желательно.Переход на единицу означает, что вы умножаете на 1 (что легко сделать).
  • Поворот по главной диагонали — это хорошо, но не обязательно. Некоторым людям нравится начинать с левого верхнего угла и продвигаться вниз к Нижний правый.
  • Если вы поворачиваете только один раз для каждой строки и столбца, столбцы, которые были очищены, останутся очищенными.
  • Поскольку точка поворота — очистить столбец поворота, выбор столбец, в котором уже есть нули, экономит время, потому что у вас нет чтобы изменить строку, содержащую ноль.

Выбор оси

  • Выберите столбец с наибольшим количеством нулей.
  • Использовать строку или столбец только один раз
  • Поверните на единицу, если возможно
  • Ось по главной диагонали
  • Никогда не поворачивайтесь на ноль
  • Никогда не поворачивайте вправо

Поскольку в первом ряду никого нет, у нас есть два варианта: либо мы первую строку делим на три и работаем дробями, либо делаем поворот на три и получите большие числа.Это вариант, который я собираюсь использовать. Я поверну на тройку в R 1 C 1 . Обведите его как стержневой элемент. В зависимости от вашего браузера вы элементы поворота могут быть обведены красным кружком или просто отмечены знаком * перед ним.

х y z rhs
* 3 2-4 3
2 3 3 15
5 -3 1 14

Идея состоит в том, чтобы превратить числа в рамке (желтые) в ноль.Использование комбинированного рядная операция (это не элементарная операция), это может сделать 3R 2 — 2R 1 → R 2 и 3R 3 — 5R 1 → R 3 .

Единственная строка, которая не изменяется, — это строка, содержащая элемент поворота ( 3). Весь смысл процесса поворота состоит в том, чтобы обнулить значения в рамке. Перепишите сводную строку и очистите (сделайте ноль) сводный столбец.

х y z rhs
* 3 2-4 3
0
0

Для замены значений в строке 2 каждый новый элемент получается путем умножения элемент, заменяемый во второй строке на 3 и вычитающий в 2 раза элемент в первой строка из того же столбца, что и заменяемый элемент.

Чтобы выполнить поворот, приложите один палец к оси поворота (обведено кружком). номер) и один палец на заменяемом элементе. Умножьте эти два числа вместе. Теперь поместите один палец на номере в рамке в той же строке, что и элемент, который вы заменяя и другой палец в поворотном ряду и такой же столбец как номер, который вы заменяете. Умножьте эти два числа вместе. Возьмите изделие за шарнир и вычесть произведение без оси.

х y z rhs
* 3 2-4 3
2 3 3 15
5 -3 1 14

Чтобы заменить 3 в R 2 C 2 , вы должны взять 3 (3) — 2 (2) = 9 — 4 = 5.

Чтобы заменить 3 в R 2 C 3 , вы должны взять 3 (3) — 2 (-4) = 9 +8 = 17.

Чтобы заменить 15 в R 2 C 4 , вы должны взять 3 (15) — 2 (3) = 45 — 6 = 39.

Чтобы заменить -3 в R 3 C 2 , вы должны взять 3 (-3) — 5 (2) = -9-10 = -19.

Чтобы заменить 1 в R 3 C 3 , вы должны взять 3 (1) — 5 (-4) = 3 + 20 = 23

Чтобы заменить 14 в R 3 C 4 , вы должны взять 3 (14) — 5 (3) = 42-15 = 27.

Вот как выглядит процесс.

х y z rhs
поворотный ряд, копия
3
поворотный ряд, копия
2
поворотный ряд, копия
-4
поворотный ряд, копия
3
поворотная стойка, прозрачная
0
3 (3) — 2 (2)
5
3 (3) — 2 (-4)
17
3 (15) — 2 (3)
39
поворотная стойка, прозрачная
0
3 (-3) — 5 (2)
-19
3 (1) — 5 (-4)
23
3 (14) — 5 (3)
27

Или, если убрать комментарии, матрица после первого поворота выглядит так.

х y z rhs
3 2-4 3
0 5 17 39
0-19 23 27

Пришло время повторить весь процесс.Мы проходим и выбираем другое место для поворота. Мы хотел бы, чтобы он был на главной диагонали, с единицей или с нулями в столбце. К сожалению, у нас не может быть ничего из этого. Но так как мы должны все умножить другие числа у оси, мы хотим, чтобы она была маленькой, поэтому мы перейдем к 5 дюймов R 2 C 2 и очистите 2 и -19.

х y z rhs
3 2-4 3
0 * 5 17 39
0-19 23 27

Начните с копирования сводной строки (2-я строка) и очистки сводного столбца (2-я строка). столбец).Ранее очищенные столбцы останутся очищенными.

х y z rhs
0
0 * 5 17 39
0 0

Вот вычисления, чтобы найти следующее взаимодействие.Обратите особое внимание в 3-ю строку, где мы вычитаем значение -19 раз. Поскольку мы вычитаем отрицательный, я записал его как плюс 19.

х y z rhs
5 (3) — 2 (0)
15
поворотная стойка, прозрачная
0
5 (-4) — 2 (17)
-54
5 (3) — 2 (39)
-63
поворотный ряд, копия
0
поворотный ряд, копия
5
поворотный ряд, копия
17
поворотный ряд, копия
39
ранее погашено
0
поворотная стойка, прозрачная
0
5 (23) + 19 (17)
438
5 (27) + 19 (39)
876

И получившаяся матрица.

х y z rhs
15 0-54-63
0 5 17 39
0 0 438 876

Обратите внимание, что все элементы в первой строке кратны 3 и все элементы в последней строке кратны 438.Разделим, чтобы сократить ряды.

х y z rhs
5 0-18-21
0 5 17 39
0 0 1 2

Это имело дополнительное преимущество, давая нам 1, именно там, где мы хотим, чтобы он был вращаться.Итак, мы переместимся на 1 в R 3 C 3 и уберем -18 и 17. Обведите свою точку поворота и поместите остальные числа в рамку. этот столбец очистить.

х y z rhs
5 0-18-21
0 5 17 39
0 0 * 1 2

Скопируйте сводную строку и очистите сводный столбец.Ранее очищенные столбцы останется очищенным до тех пор, пока вы не повернете строку или столбец дважды.

х y z rhs
0 0
0 0
0 0 * 1 2

Обратите внимание, что каждый раз приходится выполнять меньше вычислений.Вот расчеты для этой оси. Опять же, поскольку значение в сводном столбце в первая строка -18, и мы вычитаем, я записал это как + 18.

х y z rhs
1 (5) +18 (0)
5
ранее погашено
0
поворотная стойка, прозрачная
0
1 (-21) + 18 (2)
15
ранее погашено
0
1 (5) — 17 (0)
5
поворотная стойка, прозрачная
0
1 (39) — 17 (2)
5
поворотный ряд, копия
0
поворотный ряд, копия
0
поворотный ряд, копия
1
поворотный ряд, копия
2

И получившаяся матрица.

х y z rhs
5 0 0 15
0 5 0 5
0 0 1 2

Обратите внимание, что первая и вторая строки кратны 5, поэтому мы можем уменьшить их ряды.

х y z rhs
1 0 0 3
0 1 0 1
0 0 1 2

И окончательный ответ: x = 3, y = 1 и z = 2.Вы также можете написать это как упорядоченный триплет {(3,1,2)}.

Надеюсь, вы заметили, что когда я работал над этим примером, я не следовал подсказкам Я дал. Это потому, что я хотел, чтобы вы увидели, что произойдет, если вы не повернетесь на один. В исходной матрице был один на главной диагонали, и Лучше было бы начать с этого.

Сводка

  • Разумно выбирайте поворотный элемент.
  • Выбор столбца с нулями означает меньший поворот.
  • Выбор единицы в качестве точки поворота уменьшает числа, упрощает умножение и оставляет ненулевые элементы в очищенном столбце те же (без поворота)
  • Поворот по главной диагонали означает, что вам не придется переключать строки, чтобы поместить матрицу в приведенная строчно-эшелонированная форма.
  • Не поворачивайтесь на ноль.
  • Не поворачивайте вправо.
  • Используйте строку или столбец только один раз
  • Возьмите продукт с шарниром за вычетом продукта без шарнира

Особые случаи

Если вы получите строку из всех нулей, кроме правой части, то система не имеет решения.